Upper Limb (2008-2019) Flashcards

1
Q
  1. What are 4 stabilizers of the AC joint? (2014)
A

JAAOS - AC Joint Injuries:

  • Static Stabilizers:
    • Joint capsule
    • AC ligaments
    • CC Ligaments
      • Conoid ligament (medial)
      • Trapezoid ligament (lateral)
  • Dynamic Stabilizers:
    • Anterior deltoid muscle
    • Trapezius through fascial insertion on acromion
How well did you know this?
1
Not at all
2
3
4
5
Perfectly
2
Q
  1. What is the best indication for non-operative treatment of a Pec Major injury?
  2. If the intimal fascia is intact and attached to the medial antebrachial fascia
  3. Proximal tear in the muscle
  4. No cosmetic deformity when the muscle is at rest
  5. Inferior tear of the tendon because of the spiral orientation
A

ANSWER: B

2013, 2016

JAAOS – Pectoralis Major Tears

  • Investing fascia is continuous with brachium and medial antebrachiam septum  can often be confused for an intact tendon on palpation (not option A)
  • Indications for repair:
    • Complete tears, myotendinous junction, tendon tears
  • ElMaraghy AW (JSES 2011)
    • Supports non-operative management of proximal muscle tears
    • Also, two people wrote in to JAAOS after that Pec Major tendon article was published to say they treat all their muscle tears (not avulsions) non-op, including in power lifters, and they have good results
How well did you know this?
1
Not at all
2
3
4
5
Perfectly
3
Q

3.Which of the following is true about atraumatic SC joint arthritis

  1. Common joint involved with RA
  2. Elderly people get atraumatic anterior SC dislocations without generalized ligamentous laxity
  3. Freidrich’s avascular necrosis produces irregularity and curving of the medial clavicle
  4. Carroll will for sure get it because he throws a softball like a girl.
A

ANSWER: C

2013

JAAOS 2005 - Atraumatic Disorders of the SC Joint

  • Rheumatoid arthritis has variable involvement, one study estimated 30%
  • Spontaneous anterior subluxation generally occurs in teens or twenties in patients with ligamentous laxity
  • Freidrich’s Disease
    • Aseptic osteonecrosis of the medial clavicle
    • Discomfort, swelling and crepitus of SC joint
    • Ipsilateral ROM loss
How well did you know this?
1
Not at all
2
3
4
5
Perfectly
4
Q

4.Patient with hand pustules, acne, pain and swelling at SC joint. What is the diagnosis?

  1. condensing osteitis
  2. friedrich’s
  3. sternal hyperostosis
  4. infection
A

ANSWER: C

2011

JAAOS 2005 - Atraumatic Disorders of the Clavicle

  • Sternocostoclavicular Hyperostosis
    • Rare disorder of soft-tissue ossification between clavicles associated with severe acne and palmoplantar pustulosis
    • Japanese males in 4-6th decade
  •  SAPHO – synovitis, acne, pustulosis, hyperostosis osteomyelitis
How well did you know this?
1
Not at all
2
3
4
5
Perfectly
5
Q

5.SC joint anterior dislocation. Best treatment?

  1. Closed reduction and figure of eight brace
  2. Open reduction and suture fixation
  3. Do nothing
  4. K wire fixation
A

ANSWER: A (but C is reasonable)

2011

JAAOS 2011 - Management of Traumatic Sternoclavicular Joint Injuries

  • Closed reduction is the current treatment of choice, although there is still some controversy regarding management because good long-term results have been reported with nonsurgical management
  • Patient under sedation, pressure on medial clavicle, immobilization with figure of eight brace x 6 weeks
  • Most unstable after reduction, but if they do stay there is better cosmesis
    • Do not recommend open reduction
  • Acute – closed reduction
  • Chronic – do nothing
How well did you know this?
1
Not at all
2
3
4
5
Perfectly
6
Q

6.What is the best view for AC joint?

  1. Zanca view
  2. Stryker notch
  3. AP shoulder
  4. Oblique View
  5. Outlet coracoid view
A

ANSWER: A

2012

JAAOS 2009 - AC Joint Injuries

  • Zanca View visualizes AC joint the best
  • Done with beam tilted 10-15o cephalic
How well did you know this?
1
Not at all
2
3
4
5
Perfectly
7
Q

7.The following are false regarding shoulder imaging, except:

  1. West point view is best for imaging a Hill-Sachs lesion
  2. Garth view is good for Hill-Sachs and Bankart lesions
  3. X-ray is sufficient for assessing glenoid bone stock
  4. MRI is best for assessing glenoid bone loss
A

ANSWER: B

2015

  • Probably an all are false except
  • JAAOS - Hill Sachs
    • Modified West Point axillary view –> glenoid bone loss
    • Stryker Notch View best for Hill-Sachs lesion
    • CT is a superior option for bone loss
  • JAAOS - Bone Loss
    • “MRA studies may suggest the degree of bone loss in the most lateral glenoid cut on the sagittal oblique series. However, the current standard imaging modality for quantifying glenoid bone loss is CT”
    • Garth WP (JBJS 1984) Roentgenographic demonstration of instability of the shoulder: the typical oblique projection. A technical note
    • Showed a cross-sectional projection of the lesion in the posterolateral sector of the humeral head known as the Hill-Sachs lesion. The edge of the anteroinferior margin of the glenoid projecting into that lesion was also shown
How well did you know this?
1
Not at all
2
3
4
5
Perfectly
8
Q
  1. Injury to which nerve is least likely to result in scapular winging?
    a. Upper trunk – dorsal scapular
    b. Nerve to subscapularis
    c. Long thoracic
    d. Spinal accessory
A

ANSWER: B

2009, 2015

JAAOS - Scapular Winging

  • Primary - dysfunction to serratus anterior, trapezius, rhomboids, levator
  • Secondary - intra-articular glenohumeral
  • Long thoracic = serratus anterior
  • Spinal accessory = trapezius
How well did you know this?
1
Not at all
2
3
4
5
Perfectly
9
Q
  1. A man falls off a scaffold and has neck and arm pain with initial x-rays negative in the ER (he has no fractures). He doesn’t get better and shows up in your clinic 3mth later. His exam reveals scapular winging, weak shoulder abduction, numbness on the lateral aspect of the shoulder. What is the most likely diagnosis?
  2. Long thoracic nerve palsy
  3. C6 nerve root lesion
  4. Axillary nerve palsy
  5. Upper trunk injury
  6. Posterior cord injury
A

ANSWER: B

2008, 2012, 2016

  • Scapular winging = long thoracic nerve, therefore must be at least root
  •  •C6 injury – weakness in wrist extension, diminished biceps reflex, sensation lateral forearm. Affects axillary nerve as well as long thoracic causing weakness in shoulder abduction, regimental patch numbness and scapular winging.
  • Long thoracic nerve – serratus anterior palsy, medial scapular winging. Given off at root level (prior to trunk)
  • Posterior cord – gives off upper (subscapularis) & lower (subscapularis & teres major) subscapular nerves, thoracodorsal nerve (latissimus dorsi), axillary and radial nerves. (C5-T1)
  • Upper trunk – Lateral antebrachial cutaneous nerve, continuation of musculocutaneous, from lateral cord – sensation to lateral forearm
How well did you know this?
1
Not at all
2
3
4
5
Perfectly
10
Q
  1. What is the first priority during reconstruction of a traumatic brachial plexus injury?
  2. Shoulder stability
  3. Wrist extension
  4. Protective sensation of the hand
  5. Elbow flexion
A

ANSWER: D

2013

JAAOS - Traumatic Brachial Plexus Palsy

Priority of Functional Repairs:

  1. Elbow flexion
  2. Shoulder abduction and stability
  3. Hand sensibility
  4. Wrist extension and finger flexion
  5. Wrist flexion and finger extension
  6. Intrinsic Hand Function
How well did you know this?
1
Not at all
2
3
4
5
Perfectly
11
Q
  1. Most common location of suprascapular nerve compression
  2. suprascapular notch
  3. spinoglenoid notch
  4. quadrilateral space
  5. between scalene muscles
A

ANSWER: A

  • 2011
  • Operative Techniques in Shoulder and Elbow Surgery
    • The most common site is suprascapular notch, where it is compressed by a thickened or ossified transverse scapular ligament
  • JBJS 1999 - Entrapment of the suprscapular nerve
    • “Most cases of entrapment occur when it courses under the transverse scapular ligament. The nerve then runs obliquely across the supraspinatus fossa toward the rim of the glenoid fossa and enters the infraspinatus fossa around the base of the spine of the scapula coursing beneath the spinoglenoid ligament, which is a much less common site of entrapment.”
    • Their series had 25 suprascapular notch and 7 spinoglenoid notch
    • In overhead athletes the spinoglenoid notch may be more common
How well did you know this?
1
Not at all
2
3
4
5
Perfectly
12
Q
  1. Young swimmer with shoulder pain. All are likely reasons except:
  2. Muscle imbalance
  3. Subacromial impingement
  4. Multidirectional instability
  5. Hypovascularity of the supraspinatus
A

ANSWER: D

2008

JAAOS 2016 - Swimmer’s Shoulder

  • The Painful Shoulder
  • Subacromial Impingement
  • Hyperlaxity
  • Scapular Dyskinesis (aka muscle imbalances)
  • GIRD (posterosuperior cuff fray/tear)
  • Labral Damage
  • Suprascapular neuropathy
How well did you know this?
1
Not at all
2
3
4
5
Perfectly
13
Q
  1. Dude with 3 weeks of shoulder pain; no history of trauma. Complains of pain at night and progressive weakness for the last week. MRI is normal except for diffuse edema in the infra and supraspinatus muscles. (2015)
  • What is the most likely diagnosis?
  • What test would help you determine the diagnosis?
A

What is the most likely diagnosis?

  • Parsonage Turner Syndrome

What test would help you determine the diagnosis?

Sudden onset pain, followed by flaccid paralysis

MRI:

  • Early - increased T2/edema in muscles innervated by nerves affected
  • Late - increased T1 signal –> fatty atrophy and infiltration

EMG:

  • 96% abnormal
  • Acute denervation with PSW and fibrillations at 3-4 weeks
  • May see chronic denervation at 3-4 months

Suprascapular nerve block

How well did you know this?
1
Not at all
2
3
4
5
Perfectly
14
Q
  1. What are 3 local biologic or anatomic features predictive of non-healing after rotator cuff repair? (2016)
A
  • JAAOS 2014 - Rehabilitation following arthroscopic Rotator Cuff Repair
  • “The re-tear rate is related to patient factors such as:
    • Age
    • Tear size
    • Tissue quality
    • Fatty infiltration
    • And medical comorbidities
    • …as well as to surgical factors”
  • From Jess’ CSES 2017 notes from lectures/small groups
    • Age >65 (this is not a local/anatomic factor)
    • Tear size (full thickness and involvement of >1 tendon)
    • Atrophy/fatty infiltration (Goutallier grade 3 or 4) / Tangent sign (if supra below)
    • Degree of muscle retraction (i.e. lateral tendon at level of glenoid)
    • Severely hooked acromion (type III)
How well did you know this?
1
Not at all
2
3
4
5
Perfectly
15
Q
  1. Excluding medical co-morbidities, what are 4 patient factors (not tear characteristics) that predict poor rotator cuff healing post operatively? (2013)
A

OKU 10

  • Age older than 65 years
  • Female sex
  • Smoking
  • Duration of symptoms
  • Medical comorbidities
  • Inability to elevate >100o
  • Weak elevation and external rotation
How well did you know this?
1
Not at all
2
3
4
5
Perfectly
16
Q
  1. What predicts poor outcome following rotator cuff tear?
  2. Pain with resisted external rotation
  3. Acromiohumeral distance of 1cm
  4. Small tear size
  5. Weakness in forward elevation
A

ANSWER: D

2008, 2014, 2016

JAAOS 1994 - Iannotti

  • Significant weakness of external rotation and significant muscular atrophy are associated with larger chronic full thickness tears
  • “A less favorable prognosis for functional recovery following surgery also should be anticipated in patients with the constellation of large chronic rotator cuff defects, chronic rupture of the long head of the biceps tendon, marked weakness of forward flexion, chronic atrophy of the deltoid and cephalic migration of the humeral head when active elevation of the arm is attempted”
  • Ellman H (JBJS 1986) Repair of the rotator cuff. End-result study of factors influencing reconstruction
  • The strength of abduction and of external rotation before repair was of prognostic value: the greater the weakness, the poorer the result. The poorest results were in patients with strength ratings of grade 3 or less. Limitation of active motion preoperatively was also of prognostic value: in patients who were unable to abduct the shoulder beyond 100o preoperatively, there was an increased risk of a poor result. An acromiohumeral distance of 7mm or less suggested a larger tear and likelihood that there would be less strength in flexion, less active motion, and lower scores”
How well did you know this?
1
Not at all
2
3
4
5
Perfectly
17
Q
  1. Benefit of double-row repair for rotator cuff tear:
  2. Better in large tears
  3. No difference in biomechanical outcomes
  4. No difference in clinical outcomes
  5. No difference in costs
A

ANSWER: C

2015

  • JAAOS 2014 - single-row versus double-row rotator cuff repairs
    • “double-row repair configurations for rotator cuff tears provide a superior biomechanical construct and improved footprint coverage. However, clinical studies are needed to determine whether double-row repairs provides substantially better structural healing or functional outcomes than does single-row repair”
    • “…justify the increased surgical time and expense of double row repair”
  • (JBJS 2010) Outcomes of Single Row and Double Row Arthroscopic Rotator Cuff Repair
    • No difference in clinical outcomes
    • One study shows double row better for massive tears (>3cm) compared to small-medium tears
How well did you know this?
1
Not at all
2
3
4
5
Perfectly
18
Q
  1. What is the main disadvantage of using absorbable suture anchors for rotator cuff repair?
  2. Infection
  3. Reduced pull-out strength
  4. Biological reaction
  5. Modulus of elasticity?
A

ANSWER: C

2016

JAAOS 2012 - Tissue Anchor Use in Arthroscopic Glenohumeral Surgery

  • Comparisons of the mechanical strength of bio absorbable and metal anchors have yielded mixed results; some studies report inferior biomechanical characteristics, whereas others report equivalent profiles”
  • “additional evidence of inflammatory response to bio absorbable anchors, which may lead to bone osteolysis, chondral damage and significant morbidity has also been reported”
How well did you know this?
1
Not at all
2
3
4
5
Perfectly
19
Q
  1. What is true about tendon transfers for massive rotator cuff tears?
  2. Pectoralis major transfer can be used for subscapularis deficiency
  3. A latissimus dorsi transfer is indicated for supraspinatus, infraspinatus and subscapularis deficiency
  4. When doing a pec major transfer, it should pass above the conjoint tendon
A

ANSWER: A

2016

  • Latissimus dorsi transfer for irreparable subscapularis tendon tears – MUN 2018 JSES
    • LD transfer resulted in pain relief and restoration of shoulder range of motion and function. LD transfer could be considered an effective and safe salvage treatment for irreparable subscapularis tears.
  • JAAOS - Tendon Transfers for Irreparable Rotator Cuff Tears
    • Latissimus for irreparable posterosuperior tears –> wouldn’t reach subscap
    • Subcoracoid pectoralis major transfer better approximately the force vector originally provided by the subscap
    • The pec major tendon gets transferred under the conjoined tendon for the transfer. If there is not enough space under the conjoined tendon, only a partial pec major tendon transfer can be done. The location of musculocutaneous nerve must be known because the pec major tendon should lie superficial to it (i.e. in a tunnel between the deep musc n. and the superficial conjoined tendon)
How well did you know this?
1
Not at all
2
3
4
5
Perfectly
20
Q
  1. During an extensive rotator cuff repair, the subscapularis is released using the following releases EXCEPT
  2. Superior margin released from coracoid
  3. Posterior surface is released from anterior capsule, superior glenoid neck, plexus
  4. Inferior border released from axillary nerve and circumflex vessels
  5. Anterior surface released from conjoint tendon
A

ANSWER: B

2008

JAAOS - Subscapularis Tears 2005

360o release

  • Superior margin from coracoid
  • Posterior surface from anterior capsule and scapular neck
  • Inferior border from axillary nerve and circumflex vessels
  • Anterior surface from conjoined tendon
How well did you know this?
1
Not at all
2
3
4
5
Perfectly
21
Q
  1. What are 3 ways in which a Laterjet procedure through a split subscapularis stabilizes the glenohumeral joint? (2014, 2015)
A

JAAOS 2009 - Glenoid Bone Deficiency

  • Increased glenoid bone stock
  • Capsulolabral repair (may be augmented by attachments of the released coracoacromial ligaments to the labrum)
  • Dynamic sling via transfer of conjoined tendon
How well did you know this?
1
Not at all
2
3
4
5
Perfectly
22
Q
  1. Acute posterior shoulder dislocation with large anterior humeral bone defect? What are the most appropriate surgical treatment options (2010, 2012, 2014, 2015)?
A

JAAOS 2012 Hill Sachs Lesion/JAAOS 2014 Acute Traumatic Posterior Dislocation

  • Disimpaction (must be <3 weeks old)
  • McLaughlin - isolated subscap transfer into defect
  • Modified McLaughlin - subscap transfer with LT
  • Osteochondral allograft (i.e. from cadaver humeral head usually)
  • Hemiarthroplasty/TSA
  • Humeral head re-surfacing
  • Rotationplasty (historical)
How well did you know this?
1
Not at all
2
3
4
5
Perfectly
23
Q
  1. Flexion, Adduction and internal rotation of the shoulder. Which structure is primarily responsible for posterior stability?
  2. Superior Glenohumeral and Coracohumeral
  3. Middle glenohumeral
  4. Teres minor and infraspinatus
  5. Posterior inferior glenohumeral ligament and posterior labrum
A

ANSWER: D

2013

Charles and Clay: 45-90 of GH elevation = posterior IGHL vs Arm on side (0 GH elevation) = SGHL

  • Jerk test evaluates posterior band of IGHL and posterior labrum and simulates this position
  • AAOS Comprehensive Review
    • The posterior IGHL is a primary static restraint against postero-inferior translation in internal rotation and adduction
  • Miller’s Orthopedics - Table 4-8
    • SGHL/CHL –> Forward flexion/abduction/IR –> stabilizes posterior translation
  • JAAOS 2014 - Acute Traumatic Posterior Dislocation
    • Coracohumeral ligament and superior glenohuemral ligament provide little anterior resistance, but help prevent posterior translation in the flexed, adducted, and internally rotated shoulder
    • Inferior glenohumeral ligament is the main stabilizer against posterior dislocation
    • The posterior band of the inferior glenohumeral ligament restricts posterior displacement with the arm in abduction
  • JBJS Current Concepts Review 2015 - Posterior Instability
  • The posterior band of the IGHL is the most important stabilizer in adduction, flexion and internal rotation
How well did you know this?
1
Not at all
2
3
4
5
Perfectly
24
Q
  1. What is the main stabilizer to posterior shoulder translation when the shoulder is in flexion, abduction, and internal rotation? REPEAT
  2. SGHL and Coracohumeral ligament
  3. MGHL and Arcuate ligament
  4. IGHL – Posterior Band
  5. Infraspinatus and teres minor
A

ANSWER: C

2012

  • Miller’s Orthopedics - Table 4.5
    • SGHL/CHL –> Forward flexion/adduction/IR –> stabilizes posterior translation
  • JAAOS 2014 - Acute Traumatic Posterior Dislocation
    • Coracohumeral ligament and superior glenohuemral ligament provide little anterior resistance, but help prevent posterior translation in the flexed, adducted, and internally rotated shoulder
    • Inferior glenohumeral ligament is the main stabilizer against posterior dislocation
    • The posterior band of the inferior glenohumeral ligament restricts posterior displacement with the arm in abduction
How well did you know this?
1
Not at all
2
3
4
5
Perfectly
25
Q
  1.  What is a static restraint of the GH joint?
    a. CH ligament
    b. CA ligament
    c. biceps
    d. negative intra-articular pressure
A

ANSWER: D

2015

Miller’s Orthopedics (p. 320)

  • Static Restraints:
    • Glenoid labrum, articular version, articular conformity, negative pressure, capsule, capsuloligamentous structures
  • Dynamic Restraints:
    • Joint concavity compression by cuff
  • Pagnani MJ (JSES 1994) Stabilizers of the glenohumeral joint
    • The coracohumeral ligament appeared to have no significant suspensory role
How well did you know this?
1
Not at all
2
3
4
5
Perfectly
26
Q
  1. What is not a risk factor for redislocation following Bankart repair:
  2. Contact sport
  3. Hill Sachs visible on external rotation x ray
  4. Age >40yrs
  5. Ligamentous hyperlaxity
A

ANSWER: C

2015

JAAOS 2014 - Anterior Glenohumeral Instability

Risk Factors for failure of arthroscopic capsulolabral repair:

  • Ligamentous laxity
  • Hill-Sachs > 250mm3
  • Immobilization <4 weeks
  • <28 years old
  • Glenoid bone loss > 15%
  • Contact sports

ISIS score components

  • Age at surgery
  • Competitive sports
  • Contact or forced overhead sports
  • Shoulder hyperlaxity
  • Hill-Sachs visible on ER A/P radiograph
  • Glenoid loss of contour on AP radiograph
How well did you know this?
1
Not at all
2
3
4
5
Perfectly
27
Q
  1. Regarding a Hill-Sachs lesion following anterior shoulder dislocation, which of the following is true?
  2. Located in the posterolateral aspect of the humeral head
  3. Is the most common cause of recurrent dislocation
  4. Best seen on AP view
  5. Best seen on the AP view with the humerus in maximal external rotation
A

ANSWER: A

2012, 2014

  • The Hill-Sachs lesion: diagnosis, classification, and management. JAAOS. 2012
    • “The Hill-Sachs lesion is a compression fracture of the posterosuperolateral humeral head that occurs in association with anterior instability or dislocation of the glenohumeral joint.”
    • Stryker notch view is most specific for evaluating a Hill-Sachs lesion - adduction and internal rotation view brings posterolateral aspect of humeral head into view
    • Lesions <20% of humeral head are of no clinical significance; lesions >40% are nearly always clinically significant and can result in recurrent anterior instability - basis of classification system (Flatow & Warner)
    • Consider all GH anatomy when planning treatment - small HS lesions can be clinically significant in the setting of large glenoid bone loss
    • Address primary problem prior to addressing HS lesion = Bankart or glenoid bone loss
    • Surgical options for HS lesions include auto/allograft, soft tissue filling (remplissage), disimpaction, hemiarthroplasty/resurfacing
  • Note: the ISIS score looks at whether the Hill-Sachs is visible on an AP view with the shoulder in maximal ER but this is NOT the best way to see it. IR is best, but they used ER for their score because they found that if the lesion was seen on a maximal ER view then it was located more superior on the head and therefore more likely to be engaging and associated with instability.
How well did you know this?
1
Not at all
2
3
4
5
Perfectly
28
Q
  1. Patient with a Hill Sachs lesion. All of the following are choices for surgical treatment, except:
  2. Lesser Tuberosity transfer into defect
  3. Infraspinatus transfer into the defect
  4. Allograft humeral head to fill the defect
  5. Remplissage
A

ANSWER: A

2013

  • JAAOS – The Hill Sachs Lesions
  • Surgical management:
    • Capsular Shift
    • Glenoid bone augmentation
    • Humeral head bone augmentation
      • Autograft, fresh or frozen allograft or synthetic
    • Tissue Filling (Replissage)
    • Connolly procedure = open procedure involving transfer of the infraspinatus and portion of GT into defect
    • Remplissage:
      • Converted to extra-articular defect with soft-tissue coverage
      • Arthroscopic posterior capsulodesis and infraspinatus tenodesis with fixation to the surface of the Hill sachs defect
    • Disimpaction
    • Resurfacing/Prosthesis
  • Posterior Instability:
    • McLaughlin = open transfer of the subscap tendon and LT to fill humeral head defect
How well did you know this?
1
Not at all
2
3
4
5
Perfectly
29
Q
  1. What is true about recurrent traumatic shoulder dislocation?
  2. Females have more recurrent traumatic shoulder dislocation
  3. More likely if Glenoid fracture
A

ANSWER: B

2016

How well did you know this?
1
Not at all
2
3
4
5
Perfectly
30
Q
  1. All the following are components of the rotator cuff interval EXCEPT?
  2. CHL
  3. SGHL
  4. Glenohumeral joint capsule
  5. Short head of biceps tendon
A

ANSWER: D

2014

  • OKU Shoulder and elbow 4
    • Rotator interval is the area of the capsule between the supraspinatus and subscapularis, includes the…
      • SGHL
      • Coracohumeral ligament
      • Long head of biceps
    • Contracture common in adhesive capsulitis, should release if treating arthroscopically
    • Laxity in instability, tightening is controversial
How well did you know this?
1
Not at all
2
3
4
5
Perfectly
31
Q
  1. 50 M fell and had anterior dislocation of shoulder. Reduced in ER 1 hour after dislocation. Presents to office 1 week later with full passive ROM but weak Abduction. Why?
  2. RTC tear
  3. Axillary nerve injury
  4. Frozen Shoulder
  5. C6 root injury
A

ANSWER: A

2011

  • Neviaser RJ (JBJS 1988) Concurrent rupture of the rotator cuff and anterior dislocation of the shoulder in the older patient
    • 31 patients unable to abduct arm after reduction of primary dislocation
    • All patients older than 35
    • 29 presumed to have axillary nerve injury –> only 1/24 positive on EMG
    • 7.8% axillary nerve injury, 100% rotator cuff tear
How well did you know this?
1
Not at all
2
3
4
5
Perfectly
32
Q
  1. Mechanism of action for posterior should dislocation
  2. Abduction, ER
  3. Abduction, IR
  4. Elevation, ER
  5. Elevation, IR
A

ANSWER: D

2008

How well did you know this?
1
Not at all
2
3
4
5
Perfectly
33
Q
  1. Posterior shoulder instability. All are true EXCEPT
  2. Dislocation is more common than subluxation
  3. Non operative treatment is usually satisfactory
  4. Posterior Bankart is unusual
  5. Commonly associated with multidirectional subluxation
A

ANSWER: A

2008

How well did you know this?
1
Not at all
2
3
4
5
Perfectly
34
Q
  1. Posterior shoulder dislocation is associated with. All of the following except:
  2. Pts can voluntarily dislocate their own shoulder
  3. Glenoid osteotomy is no longer indicated
  4. Surgery is often helpful
  5. Subluxation is more common than dislocation
A

ANSWER: B

2009

JAAOS 2017 – Posterior GH instability: Evidence based treatment

  • Operative open or arthroscopic posterior labral repair (Bankart) indications
  • recurrent posterior shoulder instability despite appropriate course of physical therapy
  • continued pain with loading of arm in forward flexed position (bench press, football blocking)
  • negative Beighton score
  • outcomes
  • 80% to 85% success at 5- to 7-year follow-up after open repair
  • similar outcomes with arthroscopic repair after shorter follow-ups
  • open or arthroscopic posterior capsular shift and rotator interval closureindications
  • positive Beighton score
  • posterior glenoid opening wedge osteotomyindications
  • excessive congenital glenoid retroversion
  • limited studies assessing outcomes with this approach
  • open reduction with subscapularis transfer (McLaughlin) or lesser tuberosity transfer to the defect (Modified McLaughlin) indications
  • chronic dislocation < 6 months old
  • reverse Hill-Sachs defect < 40%
  • hemiarthroplastyindications
  • chronic dislocation > 6 months old
  • severe humeral head arthritis
  • collapse of humeral head during reduction
  • reverse Hill-Sachs defect > 40% of articular surface
  • total shoulder arthroplastyindications
  • significant glenoid arthritis in addition to one of the hemiarthroplasty indications
How well did you know this?
1
Not at all
2
3
4
5
Perfectly
35
Q
  1. Give the 4 types of SLAP tears. (2011)
A

Snyder Classification:

  1. Labral fraying
  2. Fraying with detached biceps tendon anchor
  3. Bucket handle tear, intact biceps tendon anchor
  4. Bucket hand with detached biceps anchor (bucket extends up biceps)
How well did you know this?
1
Not at all
2
3
4
5
Perfectly
36
Q
  1. What are 3 mechanisms of injury for a SLAP tear? (2010, 2013, 2015)
A

JAAOS 2009 - Superior Labral Tears of the Shoulder

  • Forceful traction injury to the arm
  • Direct compression load/blow
  • Repetitive overhead throwing activities (peel back)
How well did you know this?
1
Not at all
2
3
4
5
Perfectly
37
Q
  1. List 4 ways to determine the correct height of a humeral prosthesis in a hemiarthroplasty performed for a 3-4 part fracture (2016)
A

JAAOS 2012- Hemiarthroplasty for three and four part proximal humerus fractures

  • Top of prosthesis should be 5.6cm from upper border of pec major
  • GT 10mm below articular surface
  • GT/LT fragments reduce without tension
  • GT superomedial corner 4.2cm above the pec major insertion
  • (GT should be between 10-16mm from superior margin of the humeral head, over or under reduction correlated with poor results)

Side note: should be in 20-30deg retroversion as measured from the epicondylar axis.

Can be done by placing lateral fin of prosthesis 30deg posterior to posterior edge of bicipital groove’

CORR Course:

Size/template other side

Intra-operative fluoro

How well did you know this?
1
Not at all
2
3
4
5
Perfectly
38
Q
  1. All of the following are true in patients with Adhesive Capsulitis, except ?
  2. Associated with thyroid disease
  3. Relief with corticosteroid injection
  4. Favorable natural history for resolution
  5. Impingement test will be positive
A

ANSWER: D

2010, 2012

JAAOS Adhesive Capsulitis

  • Associated Conditions:
    • Cardiovascular disease
    • Thyroid dysfunction/ACTH deficiency
    • Breast cancer treatment
    • Risk factors –> CVA, MI, DM
    • DM has worse prognosis
    • Minor trauma to limb
    • Heart Disease, cardiac surgery
    • Parkinsonism
    • Malignancy
    • Hyperlipidemia
    • Drugs –> MMP inhibitors (glaucoma), antiretroviral, pneumococcal/influenza vaccine, fluoroquinolones
    • Dupuytren’s contracture
  • Transient pain relief with corticosteroid injections
  • Natural resolution of symptoms with mild treatment
How well did you know this?
1
Not at all
2
3
4
5
Perfectly
39
Q
  1. 18 year-old female presents to you complaining of her shoulder going “in and out” when swimming or playing volleyball. Never had a traumatic dislocation. Which of the following is the best option for treatment of her instability?
  2. Bankart repair
  3. Magnuson-Stack procedure
  4. Inferior capsular shift
  5. Putti-Platt procedure
A

ANSWER: C

2012

JAAOS - Management of Multidirectional Instability of the Shoulder

Reconstructive techniques include glenoid osteotomy, labral augmentation, capsuloligamentous reconstruction

Open Inferior Capsular Shift

How well did you know this?
1
Not at all
2
3
4
5
Perfectly
40
Q
  1. Most common cause of upper extremity neuropathic arthropathy(charcot)?
  2. Syringomyelia
  3. Diabetes
  4. Syphilis
  5. Multiple Sclerosis
  6. Hansen - leprosy
A

ANSWER: A

2012

JAAOS 1996 - Neuropathic Arthropathy + Millers 8th edition p.40

  • Neuropathy in the shoulder (upper extremity) is most commonly associated with syringomyelia
  • Second most common charcot upper extremity arthropathy is Hansen-Leprosy

How well did you know this?
1
Not at all
2
3
4
5
Perfectly
41
Q
  1. Lady with mild trauma, no pain, not wearing a splint, xray shows destroyed distal radius. How do you make the diagnosis. No constitutional symptoms.
  2. EMG
  3. ESR/CRP
  4. MRI C Spine
  5. HbA1c
A

ANSWER: C (first to r/o syrinx then DM2)

2008

  • Jackson et al. 2012. Charcot arthropathy of the wrist associated with cervical spondylotic myelopathy.
  • The most frequently cited cause of Charcot arthropathy of the upper extremity is syringomyelia, and magnetic resonance imaging of the cervical spine should be obtained at presentation.
How well did you know this?
1
Not at all
2
3
4
5
Perfectly
42
Q
  1. What is not true about internal impingement of the shoulder in a throwing athlete?
  2. Results in full-thickness RC tear
  3. Treatment with sleeper stretches
  4. Tight posterior capsule
  5. Associated with mild shoulder instability
A

ANSWER: A

2014, 2016

JAAOS 2007 - Understanding Shoulder and Elbow Injuries in Baseball

  • Internal impingement has been through to have the following etiologies contributing: traction on biceps tendon, laxity of anterior band of IGHL caused by excessive external rotation stretch, posterior capsular tightness and scapular dyskinesia

JAAOS - Posterior Capsular Contracture of the Shoulder:

  • Non-operative treatment includes sleeper stretches
  • Micro-instability is associated

OKU Shoulder and Elbow:

  • Regardless of their etiology, injuries to the throwing shoulder share many final sequelae, including superior labrum anterior to posterior (SLAP) tears, proximal biceps pathology, partial thickness rotator cuff tears, and scapular dyskinesia
  • “Full thickness tears are uncommon, but partial thickness articular sided tears are almost ubiquitous in throwing athletes”
How well did you know this?
1
Not at all
2
3
4
5
Perfectly
43
Q

  43. Regarding thoracic outlet syndrome. Which of the following is true:

  1. Compression between the anterior scaline and SCM
  2. Positive Watson test
  3. Elevation of the scapula
  4. More common in males
  5. Adson Test +
A

Answer: E.

  • C is false: depression
  • D is false: women more common
  • B is false: SL instability
  • A is false: anterior and middle scalene

2008, 2014 (confirmed McGill)

2008 question has female gender which I like best

  • JAAOS 2015 - Thoracic Outlet Syndrome
    • Thoracic outlet is interval from supraclavicular fossa to axilla (between clavicle and first rib)
    • Typical patient is young, thin, female with a long neck and drooping shoulders
    • Compression:
    • Inter-scalene -anterior scalene, middle scalene, first rib
      • Subclavian artery, trunks of brachial plexus
    • Costo-clavicular - clavicle, subclavius, costocorocoid ligament
      • Subclavian vessels, divisions of brachial plexus
    • Retro-pectoralis - inferior to coracoid process
      • Brachial plexus, axillary artery and vein
  • Provocative Tests:
    • Wright - decrease in pulse with hyper-abduction and ER with head on opposite side
    • Adson - extension with head towards side + deep breath
    • Roos - 90o abduction, elbows 90o + hand open/close

“Nonsurgical management is reported to be less successful in obese patients, in patients who are on workers’ compensation, and in patients with double-crush neurologic pathology involving the carpal or cubital tunnels.”

How well did you know this?
1
Not at all
2
3
4
5
Perfectly
44
Q
  1. Thoracic outlet syndrome, which is false?
  2. Vascular cause accounts for 5%
  3. Adson’s is sensitive but not very specific
  4. Can be tested my abducting arm and internally rotating shoulder
  5. Cervical rib is most common X ray finding
A

ANSWER: C

2010

  • Wright test is symptoms with abduction an EXTERNAL rotation
  • Tests are neither sensitive nor specific (50-70% range)
  • ” Gergoudis et al1challenged the clinical utility of this test by showing that 66 of 130 normal persons (51%) had a diminished pulse with the Adson maneuver.” (JAAOS)
  • “In a series by Gillard et al. the specificity for the Adson test and for the Roos test was 76% and 30%, respectively; however, when both tests were positive, specificity increased to 82%.”
How well did you know this?
1
Not at all
2
3
4
5
Perfectly
45
Q
  1. Thoracic outlet, all of the following true except
  2. 50% vascular
  3. Occurs in females more often
  4. Can present with C8-T1 nerve symptoms
  5. First event can present with thrombosis
A

ANSWER: A

2008

JAAOS 2015 - Thoracic Outlet Syndrome

  • Vascular causes rare (3-5% venous, <1% arterial)
  • Females are more common
  • Neurogenic (90%)
  • Lower and combined plexus presentation is more common (85-90%)
  • Arterial cases –> can develop and aneurysm, thrombosis and limb threatening ischemia
How well did you know this?
1
Not at all
2
3
4
5
Perfectly
46
Q
  1. Rate of return to play at same level following SLAP repair:
  2. 5%
  3. 20%
  4. 50%
  5. 80%
A

ANSWER: C

2015

  • Brockmeier SF (JBJS 2009) Outcomes after arthroscopic repair of type II SLAP lesions
    • 47 patients at 2 years following arthroscopic SLAP repair with suture anchors
    • Return to sport (same level) = 74%
  • Neuman BJ (AJSM 2011) Results of arthroscopic repair of type II SLAP in overhead athletes
    • 30 patients at 3.5 years
    • 84% return to preinjury level of function
  • 2019 Systematic Review Return to Sport After Arthroscopic Superior Labral Anterior-Posterior Repair: A Systematic Review
    • 69% return to play at same level
How well did you know this?
1
Not at all
2
3
4
5
Perfectly
47
Q
  1. What is a cause of glenoid loosening in total shoulder arthroplasty in rotator cuff arthropathy?
    a. superior eccentric wear
    b. concentric wear
    c. posterior eccentric wear
A

ANSWER: A

2015

JAAOS 2007 - Rotator Cuff Tear Arthropathy

  • “Patients with cuff deficiency demonstrated significantly greater superior migration of the humeral prosthesis on the glenoid component compared with patients with no rotator cuff deficiency. The investigators suggested that this superior displacement of the humeral head on the glenoid led to loosening of the glenoid component and a “rocking horse” phenomenon of glenoid loosening”
  • Matsen, F (JBJS 2008) Glenoid component failure in total shoulder arthroplasty
  • As a special type of glenohumeral instability, the superior subluxation seen with massive rotator cuff deficiency is an important cause of eccentric loading, creating the risk of rocking-horse loosening of the glenoid component
How well did you know this?
1
Not at all
2
3
4
5
Perfectly
48
Q
  1. Most common cause of anterior shoulder instability after TSA?
  2. humeral anteversion
  3. glenoid anteversion
  4. subscap repair failure
  5. Posterior capsule not released
A

ANSWER: C

2011

Bohsali KI (JBJS 2006) Complications of TSA

  • Anterior and superior instability accounted for 80% of instability cases
  • Anterior instability is associated with humeral component malrotation, anterior glenoid deficiency, anterior deltoid muscle dysfunction, failure of the subscapularis tendon and anterior aspect of the capsule

Moeckel (JBJS 1993)

  • 100% of shoulders with anterior dislocation had subscap insufficiency (granted only 7 shoulders)
How well did you know this?
1
Not at all
2
3
4
5
Perfectly
49
Q
  1. What is a cause of poor outcomes for shoulder hemiarthroplasty?
  2. Rotator Cuff Tear
  3. Non concentric wear on glenoid
  4. Eburnated bone on glenoid
  5. Circumferential capsular release
A

ANSWER: B

2008

  • Levine WN (JSES 1997) Hemiarthroplasty for glenohumeral osteoarthritis: Results correlated to degree of glenoid wear
    • 31 shoulders
    • 86% satisfaction with concentric shoulders, 63% with non-concentric glenoids
  • Ianotti JP (JBJS 2003) Influence of preoperative factors on outcome of shoulder arthroplasty for glenohumeral osteoarthritis
    • 128 shoulders at 46 months
    • 10% had supraspinatus tear - no effect on ASES, decrease in pain or satisfaction
    • Patients with posterior subluxation of humeral head had worse outcomes
  • Levine WN (JBJS 2012) long term follow up of shoulder hemiarthroplastyfor glenohumeral osteoarthritis
    • 25% satisfaction at 17 years
    • Concentric glenoids and primary OA do better than non-concentric glenoids and secondary arthritis
How well did you know this?
1
Not at all
2
3
4
5
Perfectly
50
Q
  1. What is not a cause of medial elbow pain in a throwing athlete:
  2. Ulnohumeral arthritis
  3. Cubital tunnel syndrome
  4. Valgus extension overload
  5. Flexor-pronator tendonitis
A

ANSWER: A

2013, 2015

JAAOS 2001 - Medial Elbow Problems in the Over-head Throwing Athlete

  • Valgus instability
  • Valgus Extension Overload
  • Medial Epicondylitis (flexor-pronator mass)
  • Ulnar Neuropathy:
    • “athletes with ulnar neuropathy usually present with intermittent medial elbow pain that may occasionally radiate down the medial aspect of the forearm into the hand”
How well did you know this?
1
Not at all
2
3
4
5
Perfectly
51
Q
  1. Which ligament is the main stabilizer during early acceleration?
  2. Posterior band of MCL
  3. Anterior band of MCL
  4. LCL
  5. Flexor pronator mass
A

ANSWER: B

2008

How well did you know this?
1
Not at all
2
3
4
5
Perfectly
52
Q
  1. Question about acceleration phases of throwing. What had tensile forces, and what had compressive forces
  2. tension force across ulna/troch and tension across radio-cap
  3. tension across radio-cap and compression across ulna/troch
  4. compression force across ulna/troch and compression across radio-cap
  5. Tension across ulnotrochlear and compression across radiocapitellar joint
A

ANSWER: D

2009

How well did you know this?
1
Not at all
2
3
4
5
Perfectly
53
Q
  1. A patient shows up in your office with an acute distal biceps tendon rupture. He is a big baby and doesn’t want surgery. What would you advise him regarding the non-operative management of distal biceps tendon ruptures? 
  2. Supination strength will decrease 40%
  3. Elbow flexion strength will decrease 50%
  4. Pronation will be more affected than supination
  5. He is at high risk of developing shoulder pain
A

ANSWER: A

2016

Morrey (JBJS 1985) Rupture of the distal tendon of the biceps brachii. A biomechanical study

3 patients !! 40% supination strength, 30% flexion strength loss

Freeman (JBJS 2009) Nonoperative treatment of distal biceps tendon rupture compared with a historical control group

18 patients

12% flexion loss, 26% supination strength

All patients returned to their jobs at average of 12 weeks with no restrictions

Nesterenko (JSES 2010) Elbow strength and endurance in patients with a ruptured distal biceps tendon

No difference in 9 patients between injured arm and uninjured arm for either flexion or supination

How well did you know this?
1
Not at all
2
3
4
5
Perfectly
54
Q
  1. Both 1 and 2 incision techniques for distal biceps tendon repair have been described. Which is NOT true
  2. PIN can be injured in both
  3. 1 incision leads to 20% less supination strength
  4. Two incision has a higher rate of HO
  5. LACN can be injured in both techniques equally (McGill - one incision has higher LACN injury)
A

ANSWER: B, C and D

2013

  • Grewal R (JBJS 2012) Single versus double-incision technique for the repair of acute distal biceps tendon ruptures
  • RCT of male patients with 47 single incision repairs with suture anchors, 44 double incision with trans-osseous drill holes
  • No difference in supination/pronation strength
  • 10% increase in flexion strength in double incision (104% vs 94%)
  • More LACN injury with single incision (19 vs 3)
  • Majority of patients had HO prophylaxis with indomethacin
How well did you know this?
1
Not at all
2
3
4
5
Perfectly
55
Q
  1. What is the most common nerve injury in a two incision distal biceps tendon reconstruction? (LACN NOT an option)
  2. MCNF
  3. Median nerve
  4. PIN
  5. AIN
A

ANSWER: C

2012

Argument: most common nerve injuries are LACN, SRN and PIN

Other options not common

  • Kelly EW (JBJS 2000) Complications of Repair of the Distal Biceps Tendon with the Modified Two-Incision Technique
  • Retrospective review of 78 two incision repairs
  • Neuropraxias:
  • 3 LACN
  • 2SRN
  • 1 PIN
How well did you know this?
1
Not at all
2
3
4
5
Perfectly
56
Q
  1. Which of the following is the most common complication after a distal biceps reconstruction?
  2. PIN injury
  3. Lateral antebrachial cutaneous nerve injury
  4. HO
  5. Symptomatic elbow flexion contracture
A

ANSWER: B

2012

JAAOS 2010 - Distal Biceps Tendon Rupture

“Lateral antebrachial cutaneous nerve paresthesia is the most common complication of distal biceps repair and is usually the consequence of aggressive retraction”

How well did you know this?
1
Not at all
2
3
4
5
Perfectly
57
Q
  1. What is true regarding the biceps tendon insertion on the radial tuberosity?
  2. The short head is more lateral and the long head is more medial.
  3. The short head is more medial and the long head is more lateral.
  4. The long head is more proximal and the short head is more distal.
  5. The long head is more distal and the short head is more proximal.
A

ANSWER: C

2014, 2016

  • Eames (JBJS 2007) Distal biceps tendon anatomy: a cadaveric study
  • The distal biceps tendon: footprint and relevant anatomy. Journal of Hand Surgery. 2007
  • Biceps musculotendinous unit rotates 90deg from origin to insertion
  • Long head inserts proximally and short head inserts distally - both insert on the very ulnar ridge of the radial tuberosity (however, the long head is slightly more radial… which makes the short head a stronger supinator)
  • Overall footprint is 23mm distal to the articular margin of the radial head
How well did you know this?
1
Not at all
2
3
4
5
Perfectly
58
Q
  1. 50 yo construction worker has 4 months shoulder pain, then pop! Now he has a lump in his distal bicep. What to do?
  2. Repair
  3. Transfer coracobrachialis
  4. Physio for strengthening
  5. Physio for shoulder / elbow ROM
A

ANSWER: D

2009

Old Answer Summary:

Loss of supination strength of 10-20% in old studies

No difference of flexion strength in chronic tears, 10% supination strength loss

How well did you know this?
1
Not at all
2
3
4
5
Perfectly
59
Q
  1. List 4 long term complications of radial head fractures. (2011, 2013)
A
  • Elbow stiffness
  • Radiocapitellar arthrosis
  • Loss of pronation/supination
  • Longitudinal forearm instability
  • Weaker:
  • Infection
  • PIN injury
  • HO/Synostosis
  • Elbow Instability
How well did you know this?
1
Not at all
2
3
4
5
Perfectly
60
Q
  1. What are 6 risk factors that will lead to radioulnar synostosis following a both bones forearm fracture. (2010, 2011, 2013, 2014)
A

JAAOS 2012 - Management of Post-traumatic Radioulnar Synostosis

  • Risk factors:
  • Injury:
  • Proximal Third
  • Fractures at the same level
  • Comminution of both bones
  • Severe local soft-tissue injury
  • Interosseous membrane injury
  • Head Injury
  • Surgical Technique:
  • Single Incision/Boyd Approach
  • Delayed Surgical Management
  • Violation of IO membrane
  • Retained bone fragment in IO space
  • Hardware into IO membrane
  • Primary onlay bone grafting
How well did you know this?
1
Not at all
2
3
4
5
Perfectly
61
Q
  1. List 3 ways to assess implant size while doing a radial head replacement. (2012, 2014)
A
  • Compare to fracture fragments (Alolabi JSES 2013)
  • Compare to contra-lateral side (Athwal JBJS 2011)
  • Ulnohumeral Symmetry (Doornberg JHS 2006)
  • PRUJ <1mm proximal to lateral edge of coronoid (Dorrnberg JHS 2006)
62
Q
  1. List 4 technical considerations in fixing a coronoid fracture (CT showing an anteromedial facet and base coronoid fracture) - (2016)
A

JAAOS 2015 - Complex Elbow Dislocation, JAAOS - Proximal Ulna #

  • Skin incision
  • Median approach vs fixation through radial head or olecranon
  • Stable Fixation
  • Possible Ligament repair
  • ? Ulnar nerve transposition
  • Same article, another thought on points
  • Fix coronoid
  • Fix/replace radial head
  • Safe zone for hardware between Lister’s tubercule and radial styloid
  • Repair LCL
  • Suture anchors or through drill holes in lateral epicondyle
  • Tie with elbow at 90degrees and pronated
  • Potentially repair MCL
  • (this was listed as steps 1-4 in the article)
63
Q
  1. Describe the push-up test for PLRI (2014)
A
  • Prone, elbows flexed to 90 deg, shoulders abducted, forearms supinated
  • Pt attempts elbow extension
  • Positive test is pt apprehension or frank subluxation/dislocation of radial head
64
Q
  1. Patient sustained an elbow dislocation that was reduced but remained unstable at 45 degrees with posterolateral instability (2010)
  • list 3 stabilizers to posterolateral instability
  • What is the most important of these lateral structures
A

list 3 stabilizers to posterolateral instability

  • Lateral Collateral Ligament
  • Radial Head
  • Coronoid (>30-50% loss)

What is the most important of these lateral structures

  • LCL
65
Q
  1. 45 yr old male with pure ligamentous instability after elbow dislocation on lateral side. What is the management of this elbow? (2010)
A
  • Immobilize in 90o with arm in pronation
  • Follow on weekly basis /with radiographs to ensure congruent reduction
  • Hinged elbow brace at 2 weeks with extension blocks
  • Progressive ROM until full extension
  • Discontinue brace at 6-8 weeks
66
Q
  1. After performing a radial head replacement, the elbow is still unstable when in supination. All the following are potential reasons except:
  2. Radial head implant is too large
  3. LUCL is avulsed off of lateral epicondyle
  4. Coronoid fracture that has not been fixed
  5. MCL rupture that was not recognized
A

ANSWER: A

2013

  • JAAOS 2007 - Fractures of the Radial Head and Neck
  • Overstuffing the radial capitellar joint has a pressure effect on the capitellum, leading to increased forces and early wear of capitellum
  • Overstuffing leads to decreased ROM and lateral elbow pain post-op
  • Too small of radial head will lead to instability
67
Q
  1. Elbow injury with large coronoid fracture and comminuted radial head. What is not part of the usual surgical plan?
    a. Repair LUCL
    b. ORIF coronoid
    c. Repair or replace the radial head
    d. Repair MCL
A

ANSWER: D

2015

  • Pugh DMW (JBJS 2004) Standard surgical protocol to treat elbow dislocations with radial head and coronoid fracture
  • “protocol included fixation or replacement of the radial head, fixation of the coronoid fracture and repair of associated capsular and lateral ligamentous injuries, in selected cases repair of MCL”
68
Q
  1. What is the most true about elbow dislocations?
  2. Coronoid fractures of 50% height does not increase instability
  3. LUCL often tears from the epicondylar origin
  4. Medial side of elbow fails first
A

ANSWER: B

2013

JAAOS 2004 - Posterolateral Rotatory Instability

  • Lateral side goes first
  • 3 stages of soft tissue disruption (Circle of Horii)
  • LUCL –> PLRI
  • LUCL + posterior capsule –> perched elbow
  • LUCL + capsule + MCL –> anterior band and flexors intact
  • JAAOS - Coronoid Fracture
  • As the elbow dislocates, the lateral collateral ligament is torn from is humeral origin
69
Q
  1. What is the most common complication following an elbow dislocation?
  2. Lack of terminal extension
  3. Nerve Injury
  4. Heterotopic ossification
  5. Late Ligament Instability
A

ANSWER: A

2013

  • JAAOS 1996 - Acute and Chronic Instability of the Elbow
  • Recurrence is uncommon (residual instability 2%)
  • 15% rate of stiffness (>30o extension lost)
  • Myositis Ossificans is uncommon in uncomplicated dislocations (<5%)
70
Q
  1. Regarding elbow dislocation, what is true:
  2. An LCL injury should be rehabilitated in supination
  3. Should be immobilized for at least 3 weeks
  4. Often associated with posterior capitellar impaction
  5. Instability is a common complications
A

ANSWER: C

2015

Supination shortens extensor-supination mass –> want to elongate this to provide stability that LCL is lacking

3 weeks too long for immobilization, generally only 1-2 weeks

Stiffness more common than instability

71
Q
  1. Posteromedial rotatory instability what is true?
  2. Associated with radial head fracture
  3. Rarely have an LCL tear
  4. Leads to ulnohumeral arthritis
  5. Anteromedial facet of coronoid is intact
A

ANSWER: C

2014, 2016

  • Doornberg (JBJS 2006) Fractures of the anteromedial facet of the coronoid process
  • Several patterns of injuries involved:
  • Varus subluxation
  • Associated avulsion of LCL common, fracture of olecranon
  • Posterior dislocation:
  • Small anteromedial facet coronoid fractures in two
  • Three had associated radial head fractures (terrible triad)
  • Journal of Shoulder and Elbow Surgery 2014 - Strategic approach to O’Driscoll Type 2 anteromedial coronoid facet fracture
  • JAAOS 2008 Coronoid Process Fracture
  • An anteromedial coronoid fracture…mechanism is varus and posteromedial rotation force on the forearm…results in LCL rupture and then compression of medial coronoid on medial trochlea
  • “By varus force in forearm pronation, the LUCL injury and the AMCF fracture occur, but it may not be evident on a simple radiograph and can be mistakenly overlooked. If it is not detected and appropriate treatment is not carried out in the early stages, post-traumatic arthritis occurs relatively quickly, and its outcome is known to be poor.” 

Difficult elbow fractures: pearls and pitfall. Instructional course lectures. 2003

  • Posteromedial varus instability of the elbow is associated with an O’Driscoll type II coronoid process fracture that involves the anteromedial facet
  • Occurs due to a posteromedial varus rotatory force to the elbow (vs more common posterolateral rotatory force), associated with injuries to the posterior band of the MCL and lateral collateral ligament; radial head fracture and injuries to the anterior band of the MCL are rare
  • Recommended treatment is direct medial exposure with fixation of the coronoid (buttress plate for larger fragments or sutures for smaller/rim fragments) + lateral collateral ligament repair
  • If untreated, can lead to elbow instability + early ulnohumeral OA (due to altered biomechanics and joint loading) 
72
Q
  1. What structure is most important in acute posterolateral instability?
  2. MCL
  3. Coronoid
  4. Olecranon
  5. Radial head
A

ANSWER: B best answer (King)….but D is also very important.

2011, 2012, 2013

  • McGill Variant of this question has Anterior capsule instead of radial head
  • Deutch (11) –> LCL repair improved laxity without radial head replacement or coronoid fixation
  • LCL + RH improved stability even more
  • Scheeberger (8)
  • Removal of radial head and 30% of coronoid process dislocated elbow at 60o flexion
  • Stabilized just with fixation of RH
  • Defect of 50% of coronoid and RH could not be stabilized by RH alone
  • JAAOS 2009 - Posterolateral Rotatory Instability
  • Coronoid process is an important elbow stabilizer in response to axial, posteromedial and posterolateral forces
  • Morrey BF (CORR 1991)
  • Simulated VALGUS injury
  • With residual instability AFTER LCL and RH fixed, then repair of MCL more effective than fixation of small coronoid fractures
  • “The radial head is a secondary constraint to resist valgus and rotatory laxity and is significant only if the anterior bundle of the MCL is absent”
73
Q
  1. A patient has a radial head fracture. 2 fragments. What is the best management plan?
  2. ORIF with screws and/or plate
  3. CRPP
  4. Replacement
  5. Resection
A

ANSWER: A

2016

  • Ring D (JBJS 2002) ORIF of fractures of the radial head
  • Review of 56 patients at 2 years with type 4 radial head #
  • Most patients with Mason III fractures with >3 fragments had poor results (13/14)
  • 12 patients with < 3 –> no non-union, early failure or severe loss of ROM
  • Greater than 3 fragments should be excised or replaced
74
Q
  1. All are true regarding an anteromedial coronoid fracture, except:
  2. Lateral collateral ligament disruption
  3. Can occur in a terrible triad injury
  4. Joint dislocation
  5. A locking plate is recommended
A

ANSWER: D

(posteromedial instability not usually associated with terrible triad, no radial head fracture so B also right)

2012, 2016

  • JOT 2018 – canadian paper - https://pubmed.ncbi.nlm.nih.gov/30339648/
  • Conclusion: Fixation of anteromedial coronoid fractures (type 2, subtype 3) is best achieved with a plating technique. Although LPs had greater stiffness, they did not offer any advantage over conventional non-LPs with respect to fracture fragment displacement in this study.
  • Newer research: Plate, but not locking plate
  • Manitoba/Toronto Answer Variation: (in this case locking plate more clearly correct)
  • Locking plate fixation preferred
  • Can be repaired with screws
  • MCL is attached to this fragment
  • Must be treated operatively
  • JAAOS 2008 – Coronoid Fractures:
  • “coronoid tip fracture is often seen in conjunction with an elbow dislocation and radial head fracture – an injury referred to as a terrible triad “
  • Anteromedial Coronoid Fracture:
  • In an anteromedial coronoid fracture, the mechanism of injury is in the opposite direction of that of a tip fracture. A varus and posteromedial rotation force  rupture of LCL from humeral origin. As the LCL gives way the medial coronoid is forced against the medial trochlea.
  • Doornberg JN (JBJS 2006) Fracture of the Anteromedial Facet of the Coronoid Process
  • “Several patterns of injury were observed in associated with fracture of the anteromedial facet of the coronoid process. These were separated into fractures associated with either varus subluxation or complete posterior dislocation of the elbow”
  • 3 had an associated fracture of the radial head (a so called terrible triad injury)
  • JAAOS 2015 - Terrible Triad
  • “This results in an anteromedial facet fracture with associated disruption of the LCL due to a varus force. The radial head is usually not fractures
  • in a varus posteromedial instability pattern, and it is therefore by definition not a true terrible triad injury”
75
Q
  1. What is the most common type of elbow instability:
  2. Posterolateral instability
  3. Posteromedial varus instability
  4. Valgus
  5. Anterior
A

ANSWER: A

2015

  • JAAOS - Acute and Chronic Instability of the Elbow
  • “acute tear of the MCL is the most frequent isolated ligamentous injury of the elbow”
  • Confirmed at CORR Recon
  • JBJS BR 2008 Posterolateral rotator instability of the elbow
  • “Posterolateral rotatory instability is the most common type of symptomatic chronic instability of the elbow”
76
Q
  1. Elbow dislocation. No fracture. Concentrically reduced, but opens in valgus. Treatment?
  2. Sling
  3. Splint at 90 with forearm in neutral
  4. Splint at 90 with forearm in supination
  5. Splint at 90 with forearm in pronation
A

ANSWER: C

2009

I think this is dumb because the muscle masses are probably torn off anyhow. Regardless.

AAOS - Simple Elbow Dislocation

Point thumb away from injured ligaments

77
Q
  1. Elbow dislocation with no fracture. Unstable in valgus stress? What is the best management?
  2. hinged external fixation to protect the ligament and allow early ROM.
  3. splint at 90 in supination for 1-2 weeks
  4. splint at 90 in supination for 3-4 weeks
  5. fix mcl
A

ANSWER: B

2011

Sheps/Hildebrand/Boorman (Hand Clin 2004)

In the setting of a simple dislocation with incompetent unstable valgus stress, medial side (MCL) disruption then treat with splint in 90o in supination for 1-2 weeks, then ROM

78
Q
  1. What is the position of the arm when examining for PLRI
  2. Extension, valgus and supination
  3. Flexion, varus and pronation
  4. Extension, valgus and pronation
  5. Flexion, varus and supination
A

ANSWER: A

2008

Posterolateral Rotatory Instability of the Elbow: Part II. Supplementary Examination and Dynamic Imaging Techniques

To perform the lateral pivot-shift test, as shown on the left elbow of the patient in the supine position, the arm is held securely by the examiner over the patient’s head. The arm is moved from an extended to a flexed position, while the forearm is supinated and axial compression and a valgus load are applied to the elbow. As the subluxation increases, the supination torque is reduced while increasing the valgus torque. The pivot shift tends to occur between 20° and 40° of flexion, but it should be assessed at multiple angles.

79
Q
  1. Radial Neck # with 1cm shortening, most likely outcome?
  2. Wrist pain
  3. OA
  4. PIN palsy
  5. Myositis ossificans
A

ANSWER: A

2008

Longitudinal instability of IO membrane

80
Q
  1. An elderly woman has severe destructive changes of both elbows. She undergoes bilateral elbow fusion. What are the optimal positions?
  2. 45 and 120 degrees
  3. 65 and 110 degrees
  4. 90 and 120 degrees
  5. 90 and 90 degrees
A

ANSWER: B

2011, 2014

JAAOS. The elbow and its disorders, Morrey

  • “If bilateral arthrodesis is performed, one elbow should be fused in 90deg of flexion for personal care and hygiene functions and the other in 45 to 65deg of flexion for other uses. Functionally, a limited range of 100deg in mid-position is enough for most daily activities.”
  • Orthopedics 2014 - Elbow arthrodesis, a novel technique and review of the literature
  • For both sexes, 90° to 110° seems to be better for personal hygiene, whereas 40° to 70° is better for extrapersonal needs and activities. In the case of bilateral elbow arthrodesis, it is generally recommended that the dominant arm be fused at 110° and the nondominant arm at 65°
81
Q
  1. What is not true about elbow OA?
  2. Weakness
  3. Loss of Joint Space
  4. Tight Capsule
  5. Hypertrophic Osteoarthritis
A

ANSWER: B

2010

JAAOS 2008

Characterized by the relative preservation of the articular cartilage and maintenance of joint space, but with hypertrophic osteophyte formation and capsular contracture

OA at the elbow is characterized by pain, stiffness, mechanical symptoms and weakness

82
Q
  1. RA and elbow pain, with synovitis and preserved joint space, what is true regarding synovectomy?
  2. Improves ROM
  3. Needs radial head resection
  4. Good pain relief in 80% of patients over 3 years
A

ANSWER: C

2008

  • Memory obviously an issue, perhaps an except question where B is the answer
  • JAAOS 2003 - Surgical management of rheumatoid elbow
  • Synovectomy with or without radial head excision is well-recognized and accepted form of treatment
  • When done early, may be an effective treatment, lasting >10 years in up to 80% of patients
  • Pain relief is common, but the rate of recurrence increases after 5 years
  • Even in patients with advanced disease, synovectomy and capsular release with or without radial head excision may result in acceptable pain relief and increased elbow ROM
  • Radial head excision may improve forearm rotation and reduce impingement…however can lead to instability
83
Q
  1. What is true regarding triceps tendon rupture:
  2. More common in younger women than older women
  3. Typically avulses from the bony insertion
  4. Will have weakness of flexion and extension
  5. A complete tear will result in complete loss of extension
A

ANSWER: B

2015

JAAOS 2010 - Distal Triceps Rupture

  • Most common in young men
  • May have elbow extension despite complete tear
  • Weakness of extension, not flexion
  • “regardless of the mechanism of injury, triceps tendon ruptures are usually seen at the osseous insertion”
84
Q
  1. During elbow arthroscopy, which portal is most common for most common sensory nerve injury
  2. Posterolateral
  3. Anteromedial
  4. Proximal medial
  5. Proximal lateral
A

ANSWER: C

2013

  • JAAOS - Elbow Arthroscopy
  • Many of the attenuated portals described in the literature place surrounding NV structures at a higher risk than do the portals described above (ie, midlateral, proximal lateral, posterolateral, transtriceps)
  • The most common nerve injury is the ulnar>SRN>PIN>AIN
  • Kelly EW (JBJS 2001) Complications of Elbow Arthroscopy:
  • 5 ulnar nerve palsy, 1 PIN, 1 SRN, 1MABC
      • JAAOS 2014 - Elbow Arthroscopy: Indications, Techniques, Outcomes, and Complications
    • Published reports document injury to all major peripheral nerves about the elbow. In one series of 473 elbow arthroscopies, the ulnar nerve was most commonly involved, followed by the superficial radial nerve, posterior interosseous nerve, anterior interosseous nerve, and medial antebrachial cutaneous nerve, respectively.13,21,23-25 Nerve injury is more common in patients with rheumatoid arthritis because the capsule is thinned or absent, in profound preoperative contractures, and in distorted anatomy
85
Q
  1. Nerve injuries in elbow arthroscopy can be common and are devastating. All of the following are ways to minimize this risk, except:
  2. Ensuring joint is maximally insufflated with fluid at all times
  3. Drawing out surface landmarks at start of procedure
  4. Using a needle to landmark portals prior to making an incision and inserting instruments
  5. Continued use of a tourniquet at all times
A

ANSWER: D

2016

JAAOS 2008 – Elbow Arthroscopy

Complications Sections

“In every clinical case, the bony anatomy should be drawn on the patient’s elbow, an 18-guage spinal needle should be used to confirm the correct portal location before introducing the trocar and the elbow should be maximally distended at all times to displace the neurovascular structures away from the entering instruments”

86
Q
  1. Dude with elbow pain and worse with supination and resisted finger extension, what do you do?
  2. Decompress supinator
  3. Decompress brachioradialis
  4. Decompress at triceps
A

ANSWER: A

2008

  • It’s a trick! Radial tunnel syndrome
  • Dang (JHS 2008) Unusual compression neuropathies of the forearm
  • Provocative maneuvers for radial tunnel:
  • Resisted supination
  • Pain with resisted middle finger extension
  • Therefore decompress at supinator
  • Brachioradialis would be for Wartenberg (Cheiralgia)syndrome – compression at ECRL and brachioradialis, and would have pain with resisted pronation.
87
Q
  1. All are true except:
  2. Radio capitellar joint bears 20% of the load through elbow
  3. radial head fractures account for 2-4% of all fractures
  4. Radio capitellar joint is only a secondary stabilizer to valgus stress (exact wording!)
A

ANSWER: A

2008

  • JAAOS 2008 - Radial Head Arthroplasty
  • Anterior bundle of the MCL is the primary stabilizer to valgus force across the elbow
  • Radial head fractures represent 5.4% of all fractures
  • JAAOS 2007 - Radial Head Fractures
  • Radius bears 80% of load at wrist, but load sharing ratio equalizes at the elbow
  • Radiocapitellar articular accounts for as much as 60% of load transfer across the elbow
88
Q
  1. Lateral XR showing displaced coronoid base #, didn’t show radial head in the slice. 3D recon CT in coronal plane showing coronoid base #, no obvious radial head fracture. Coronoid base fracture. – APPROACH?
  2. Flexor pronator
  3. EDC
  4. Anterior
  5. Kocher
A

ANSWER: A

2019

  • a) Flexor pronator (2019)
  • No obvious radial head # - don’t need to go lateral (eliminates d and b)
  • No reason to go anterior
  • Leaves FP approach
  • Muscle-splitting approach is less traumatic to the flexor-pronator muscle mass, and it could decrease operative time and lessen immediate morbidity after surgery (compared to lifting off tendon from medial epicondyle)
  • Muscle-split through the common flexor bundle extends from the medial humeral epicondyle to a point distal to the tubercle of the ulna such that repair or reconstruction can be performed on the ulnar collateral ligament.
  • Technique
  • Standard posterior incision raising large fasciocutaneous flap
  • Site of muscle-split is through the posterior 1/3 of the common flexor bundle within the most anterior fibers of FCU
  • Principle – split muscle in safe zone between median and ulnar nerve innervated muscles
  • This is the raphe between the FCU and palmaris longus muscles superficially and the FCU and FDS muscles slightly deeper.
  • This raphe is more easily identified in the distal portion of the incision because proximally the muscles have a coalesced tendinous origin.
  • After making the fascial incision extending from the medial humeral epicondyle to a point overlying the suspected sublime tubercle (approximately 3 to 4 cm distally), the muscle is bluntly split down to the level of the UCL.
  • The muscle-split can safely be extended 1 cm distal to the UCL’s insertion on the sublime tubercle of ulna
  • This takes you right down onto UCL and also can repair coronoid fractures (King)
  • Don’t need to transpose ulnar nerve – but on exam always do this
89
Q
  1. All of the following are theories about carpal stability except:
  2. Spring theory
  3. Column theory
  4. Oval ring theory
  5. Linked theory
A

ANSWER: A

2019

  • Spring Theory
  • Column theory
  • lateral (mobile) column 
  • comprises scaphoid, trapezoid and trapezium
  • scaphoid is center of motion and function is mobile
  • central (flexion-extension) column
  • comprises lunate, capitate and hamate
  • luno-capitate articulation is center of motion 
  • motion is flexion/extension
  • medial (rotation) column
  • comprises triquetrum and distal carpal row 
  • motion is rotation
  • Oval ring theory
  • The oval-ring theory functionally depicts the carpus as a transverse ring formed by proximal and distal rows and joined by two physiologic links, one radial and the other ulnar.
  • The radial link is the mobile scaphotrapezial joint, and the ulnar link is the rotatory triquetrohamate joint.
  • (Rows concept)
  • comprises proximal and distal rows 
  • scaphoid is a bridge between rows
  • motion occurs within and between rows
  • Linked theory
  • three links in a chain composed of radius, lunate and capitate 
  • head of capitate acts as center of rotation
  • proximal row (lunate) acts as a unit and is an intercalated segment with no direct tendon attachments
  • distal row functions as unit
  • advantage
  • efficient motion (less motion at each link) 
  • strong volar ligaments enhance stability
  • disadvantage
  • more links increases instability of the chain
  • scaphoid bridges both carpal rows
  • resting forces/radial deviation push the scaphoid into flexion and push the triquetrum into extension
  • ulnar deviation pushes the scaphoid into extension
90
Q
  1. Regarding FDP injuries with retraction into the palm, all of the following are true EXCEPT:
  2. Most commonly involves the ring finger
  3. Most common type
  4. Good outcomes are expected if repair is performed within 6 weeks
  5. Often delayed diagnosis
A

ANSWER: B and C

  • not common (tpe 2 PIPJ most common) and needs sx within 7-10 days (lacuna vasorum disrupted))

2019

  • ANSWER: Good outcomes if repair within 6 weeks
  • Jersey finger = avulsion of FDP from insertion at base of distal phalanx
  • Ring finger most common – 75%
  • Type II most common type – retraction to PIPJ
  • Often delayed dx – sprain
  • Type I
  • FDP tendon retracted to the palm
  • Vascular supply disrupted
  • Prompt repair recommended – 7-10 days***
  • These injuries have a worse prognosis if not diagnosed and treated within 7–10 days as the tendon contracts and becomes less viable.
91
Q
  1. What is the treatment for a dorsal PIP fracture dislocation with 25% articular fragment. No mention of stability, comminution, or anything else.
  2. Open reduction internal fixation
  3. Dorsal block splinting
  4. Volar plate reconstruction
  5. Immobilize in 60-90 degrees of flexion
A

ANSWER: B

2019

  • Dorsal blocking splint
  • Type 1 - <30% articular surface
  • Closed reduction + splinting
  • Dorsal blocking splint
92
Q
  1. All of the following are appropriate procedures to address DRUJ instability except:
  2. Adams procedure
  3. Blatts capsulodesis
  4. Ulnar shortening osteotomy
  5. TFCC repair
A

ANSWER: B

2019

  • Blatts capsulodesis (scapholunate instability)
  • Adams procedure
  • Anatomic reconstruction of distal radioulnar ligaments
  • Ulnar shortening osteotomy
  • Resection arthroplasty
  • Matched resection
  • Darrach procedure (ulnar head resection)
  • reserved for low-demand, elderly patients
  • an unstable, painful proximal ulna stump may result
  • TFCC repair
  • Stabilizes DRUJ
  • Blatts capsulodesis –> for scapholunate instability
93
Q
  1. Which of the following is not part of the TFCC?
  2. Articular disc
  3. Ulnar collateral ligament
  4. FCU (yes, it said FCU)
  5. Volar radio ulnar ligament
A

ANSWER: C) FCU

TFCC made up of:

  • dorsal and volar radioulnar ligaments 
  • deep ligaments known as ligamentum subcruentum
  • central articular disc
  • meniscus homolog
  • ulnar collateral ligament
  • ECU subsheath 
  • origin of ulnolunate and ulnotriquetral ligaments
94
Q
  1. IF you increase the radial height by 4mm in a radial head arthroplasty, what will happen:
  2. Same consequences as if its 4mm decreased height.
  3. Increased IO membrane tightening
  4. Increased contact pressures
  5. Pronation and supination decreased by 30 decreases.
A

ANSWER: C

2019

  • Increased contact pressures
  • The pressure on the radiocapitellar joint was significantly increased after 2.5 mm of lengthening.
  • Overstuffing can lead to loss of extension – not pro/sup
  • Inc radial head height = dec ulnar variance
95
Q
  1. In regards to reverse total shoulders, All of the following have been shown to limit instability except (this is actually how it was written):
  2. Limited humeral lengthening
  3. Superior glenosphere tilt 15-20 degrees
  4. Lateralized glenosphere
  5. Inferior and eccentric placement of the glenosphere.
A

ANSWER: B

2019

  • Superior glenosphere tilt 15-20 degrees
  • Humeral lengthening – soft tissue tension
  • A 10° inferior tilt of the glenoid component in reverse shoulder arthroplasty is associated with a reduced risk of dislocation when compared to neutral tilt.
  • Factors that contribute to postoperative instability include soft-tissue tension, glenosphere diameter, constraint on the humeral socket, mechanical impingement (GT, scapular spine, coracoid process), bony deficiency, erroneous version of the prosthesis (ie, rotational position on the glenoid, NSA on humerus, version), and axillary nerve/deltoid dysfunction.
  • Treatment – increase lateral glenosphere offset, adjust NSA on humerus, adjust version, increase thickness of humeral PE, inferiorize glenoid, offset humeral component, remove impinging soft tissue (usually inferior glenoid)
96
Q
  1. The patient had an olecranon ORIF 3 months ago. She has 35-100 degrees ROM of the elbow. Normal pronosupination. Which is true?
  2. Remove the LUCL with lateral column procedure (may have said sacrifice LUCL)
  3. Loss of flexion is only due to osteophytes in the coronoid fossa ??
  4. Lymphocytic infiltration of the capsule – Myofibroblasts
A

ANSWER: C

2019

  • Prob don’t expect osteophytes 3 months after
  • Loss of joint volume = impingement of ulno-humeral on capsule (usually only affects one plane, so pro-sup can be good)
  • https://www.ncbi.nlm.nih.gov/pmc/articles/PMC5296847/?report=printable
  • JAAOS – Open surgical release for contractures of the elbow 2015
  • Extension loss is better tolerated than flexion loss.
  • Loss of elbow flexion past 110 exceeds the ability of positional adaptation of the neck/wrist/shoulder = inability to button collar, shave, place earings
  • If flexion of elbow is <100°, it requires the release of posterior band of medial collateral ligament (MCL) of elbow
  • A stiff elbow is usually defined as having less than 30 in extension or flexion of less than 130. Most activities of daily living are possible if the elbow has a range of motion of 100 (30–130 of flexion, Morreys “arc of motion”)….functional is 50-50 pro-sup
  • Aside from an elevation of the number and activation of myofibroblasts in the contracted capsule they showed that TGF-ß1 and PDGF induced a cell proliferation of MFs and increased the contraction of extracellular matrices (ECM). (Post-Traumatic Elbow Stiffness - Arthrolysis and Mechanical Distraction)
  • JAAOS 2002 – Posttraumatic elbow stiffness: Evaluation and management
  • Deficits in flexion extension are most often caused by ulnohumeral joint pathology. Unless there is HO or complete ankylosing, WILL USUALLY ONLY EXPERIENCE LOSS IN ONE FUNCTIONAL ARC , flex-ext or pro-sup
  • Soft tissue constraint has soft end point vs hard end point (bony impingement).
  • DISPLACED CORONOID FRAGMENTS OFTEN MAY BLOCK FLEXION.
97
Q
  1. What is true regarding Boutonnier deformity
  2. Lateral bands dorsally displaced
  3. Can pin the DIP
  4. Treatment includes Tenotomy over dorsal aspect of the proximal phalanx
  5. Pathology does not stem from the DIP or MCP
A

ANSWER: D (poor question…it does have DIP pathology since tenotomize terminal extensor at dorsal DIP)

2019

  • https://www.orthobullets.com/hand/6012/boutonniere-deformity
  • pathoanatomic sequence includes
  • rupture of central slip
  • causes the extrinsic extension mechanism from the EDC to be lost
  • prevents extension at the PIP joint
  • attenuation of triangular ligament
  • causes intrinsic muscles of the hand (lumbricals) to act as flexors at the PIP joint
  • lumbricals also extend the DIP joint without an opposing or balancing force
  • palmar migration of collateral bands and lateral bands
  • the lumbricals’ pull becomes unopposed, pulling through the base of the distal phalanx and volar to the PIP
  • causes PIP flexion and DIP extension
98
Q
  1. Regarding distal biceps rupture. What is the different between 1 incision and 2 incisoin technique? (seems like an outdated question, but these were the choices)
  2. Single incision associated with decreased transient neuropraxia
  3. Two incision approach associated with increased incidence of HO
  4. Shorter operative time with 1 incision technique
  5. Two-incision technique has more stiffness and worse outcomes
A

ANSWER: B

2019

  • 2016 A Meta-analysis of Single-Incision Versus Double-Incision Surgical Technique
  • Outdated since recent Grewal UWO paper says no difference in HO, just showed that single incision = more LABCN palsy
  • 10% better flexion strength in double incision
  • Single versus double incision technique for the repair of acute distal biceps tendon ruptures – 2012 Grewal et al.
99
Q
  1. All of the following can lead to Kienbochs’s:
  2. Venous outflow
  3. Clotting disorder
  4. Arterial pattern
  5. Skeletal variation
A

ANSWER: B

2019

  • https://www.orthobullets.com/hand/6050/kienbocks-disease
  • Pathophysiology
  • thought to be caused by multiple factors
  • biomechanical factors
  • ulnar negative variance
  • leads to increased radial-lunate contact stress
  • decreased radial inclination
  • repetitive trauma
  • anatomic factors
  • geometry of lunate
  • vascular supply to lunate
  • patterns of arterial blood supply have differential incidences of AVN
  • disruption of venous outflow leading to increased intraosseous pressure
  • Blood supply to capitate is also poor and may lead to AVN.
100
Q
  1. Which of the following is false?
  2. Moving valgus stress test is very sensitive for UCL injury
  3. Resisted extension of the middle finger is a way to test for lateral epicondylitis
  4. Milking maneuver has a poor negative predictive value for UCL injury
  5. Chair rise test is more sensitive than lateral pivot shift test for PLRI
A

ANSWER: C

2019

  • Maudsley test – resisted middle finger extension selectively recruits ECRB – tests lateral epicondylitis
  • Regan and Lapner31 performed a prospective evaluation of the prone push-up test, the chair push-up test, and the lateral pivot shift test; the prone and chair push-up tests demonstrated a higher diagnostic sensitivity compared with the lateral pivot shift test. For the prone and chair push-up tests, they reported 87.5% sensitivity for each test, with a combined sensitivity of 100%, compared with 37.5% for the lateral pivot shift test.
  • Milking test - The test is positive if the patient experiences a subjective feeling of apprehension and instability along with medial elbow pain. The maneuver’s sensitivity has been reported as 87.5%, with a negative predictive value of 100%.
101
Q
  1. What is true regarding Dupuytrens
  2. MCP contractures more responsive to needle aponeurotomy than PIP
  3. Limited palmar fascietcomy is more successful to reduce PIP than MCP
  4. NV bundle of the digits are not at risk with open treatment
  5. Open palm technique is associated with increased complications
A

ANSWER: A

2019

  • https://www.orthobullets.com/hand/6058/dupuytrens-disease
  • NOT involved in Dupuytren’s disease
  • Cleland’s ligament 
  • transverse ligament of the palmar aponeurosis
  • disease only involves longitudinally oriented structures
  • needle aponeurotomy
  • indications
  • mild contractures (at the MCP > PIP)
  • medical co-morbidities that preclude surgery
  • technique
  • perform in office using 22G or 25G needle
  • followed by manipulation and night orthosis wear
  • outcomes
  • more successful for MCP contracture than PIP
  • less improvement and higher recurrence rate than surgery (open partial fasciectomy)
102
Q
  1. Rotator cuff question. What is the difference between double row and single row repair? REPEAT
  2. Better functional outcomes with double row repair
  3. Double row repair biomechanically superior and better at recreating the anatomic footprint
  4. Double row is cost effective
A

ANSWER: B

2019

2015

  • JAAOS 2014 - single-row versus double-row rotator cuff repairs
  • “double-row repair configurations for rotator cuff tears provide a superior biomechanical construct and improved footprint coverage. However, clinical studies are needed to determine whether double-row repairs provides substantially better structural healing or functional outcomes than does single-row repair”
  • “…justify the increased surgical time and expense of double row repair”
  • (JBJS 2010) Outcomes of Single Row and Double Row Arthroscopic Rotator Cuff Repair
  • No difference in clinical outcomes
  • One study shows double row better for massive tears (>3cm) compared to small-medium tears
  • One of my CSES preceptors argued that although double row repairs have not been shown to improve clinical outcomes, they HAVE been shown to be associated with better healing rates (however we know that retears are not necessarily correlated with worse outcomes, so this still might not matter)
  • Arthroscopy 2011 - Single row repair versus double-row repair of full-thickness rotator cuff tears
  • Meta-analysis
  • “Despite the fact that double-row repair shows a significantly higher rate of tendon healing and greater external rotation than does single-row repair, there is no significant improvement in shoulder function, muscle strength, forward flexion, internal rotation, patient satisfaction, or return to work.”
103
Q
  1. Image given of proximal pole scaphoid fracture, pole may or may not have appeared sclerotic. Very poor quality xrays. 19M 5 months post injury. Has not received any treatment yet
  2. Volar approach and bone graft
  3. Dorsal approach and ORIF
  4. Thumb spica
  5. Scaphoid excision and partial wrist fusion
A

ANSWER: B

2019

Dorsal approach for proximal pole #s, Volar approach for waist #s.

Question depends on if it was a waste or proximal pole fracture and whether there is osteonecrosis.

104
Q
  1. What is true with respect to terrible triad injuries of the elbow?
  2. LUCL comes off the supinator crest
  3. Should always be treated with operative management
  4. MCL is often ruptured
  5. Type 3 coronoid base fracture
A

ANSWER: C

2019

105
Q
  1. Child sustains a transverse laceration 1 mm proximal to the distal crease of the little finger. He could not actively flex his DIP. With the long and ring finger extended, could not actively flex his ring finger PIP. Even with his ring finger flexed, there is still no change (the small finger cannot flex). What is true?
  2. Congenital absence of FDS to ring and small finger
  3. Laceration of small finger FDP and congenital absence of FDS
  4. Laceration of small finger FDP and FDS
  5. Laceration of small finger FDP and FDS intact
A

ANSWER: B

One maneuver is the Expanded Baker Modified Exam, where all fingers are held in extension by the examiner except the digit of interest, and then the examinee is asked to flex the PIP and DIP joints independently to test the FDS and the FDP, respectively

106
Q
  1. All of the following are true with respect to managing a walch B2 glenoid (specifically mentioned) in osteoarthritis of the shoulder except:
  2. A reverse total shoulder is an acceptable option in an intact rotator cuff
  3. Best implant survival is predicated on placing the glenoid in retroversion to avoid reaming subchondral bone
  4. It is associated with increased posterior bone loss
  5. It is associated with increased posterior humeral subluxation
A

ANSWER: B

2019

107
Q
  1. Which of the following is not an option for a vascularized graft for a scaphoid non-union
  • Volar carpal pedicled graft
  • Iliac crest free flap/graft
  • 1st dorsal metacarpal artery island flap
  • free medial femoral condyle
A

ANSWER: C

2019

  • Dorsal grafts may be better suited for management of scaphoid nonunions that involve the proximal pole and those without significant humpback deformity because adequate correction of the deformity often
  • requires a separate volar approach and increased soft-tissue dissection.
  • The use of VBG harvested from the radius and based on the volar carpal artery has also been described. Gras and Mathoulin17 reported on 38 persistent scaphoid nonunions (with no evidence of ON) following previous surgical management with a volar vascularized radius bone graft.
  • Also Ulnar Bone Grafts - corticocancellous graft is harvested with its periosteum from the medial distal third of the ulna, with the ulnar artery serving as the pedicle.
  • Thumb metacarpal vascularized bone graft in long-standing scaphoid nonunion–a useful graft via dorsal or palmar approach: a cohort study of 24 patients.
  • J Hand Surg Am. 2004 Nov;29(6):1089-97.
108
Q
  1. Reconstruction of SL insufficiency (of the wrist), which of the following cannot be used for reconstruction
  2. SLAM (scapho lunate axial method)
  3. Radio-scapho-lunate tenodesis -
  4. Tri-ligament tenodesis
  5. Scapho-luno-triquetral tenodesis
A

ANSWER: B

2019

A, C, D described

Confirmed with Suh

109
Q
  1. 35yo lady with Traumatic amputation at the middle phalanx of the long finger, was debrided and adequately closed (no mention of re-implantation), with no post op complications. 3 months later was not able to fully flex any of her fingers, when trying to make a fist. What is the most appropriate treatment:
  2. division of the extensor hood of the injured finger
  3. FDP release of the finger
  4. Aggressive physiotherapy to improve the power of the fingers
  5. Stellate ganglion block for ROM exercises
A

ANSWER: B

2019

  • The quadriga effect is characterized by an active flexion lag in fingers adjacent to a digit with a previously injured or repaired flexor digitorum profundus tendon.
  • Mechanism
  • most commonly caused by a functional shortening of the FDP tendon due to
  • over-advancement of the FDP during tendon repair
  • >1 cm advancement associated with quadriga
  • adhesions
  • retraction of the tendon
  • “over-the-top” FDP repair of the distal phalanx after amputation
  • Pathoanatomy
  • FDP tendons of long, ring, and little fingers share a common muscle belly
  • therefore excursion of the combined tendons is equal to the shortest tendon
  • improper shortening of a tendon during repair results in inability to fully flex adjacent fingers
  • Symptoms
  • inability to fully flex the fingers of the hand adjacent to the injured finger
  • patient may complain of “weak grip”
  • Physical exam
  • upon making a fist the fingers adjacent to the injured digit will not reach full flexion
  • grip strength decreased
  • Treatment
  • Release FDP of injured digit
110
Q
  1. DISI question, all true except?
  2. Scapholunate ligament disruption
  3. Scaphoid supinates
  4. radiolunate incongruent
  5. lunate extends
A

ANSWER: B or C (better is C…newer data, suggest RL stays congruent)

  • SCAPHOID PRONATES*** later stages supinates??
  • mentioned a couple times in Green’s that the scaphoid definitely Pronates in SL injury
  • Current status of scapholunate ligament injuries. JAAOS. 2002.
  • DISI = dorsal intercalated segment instability; radiographic finding defined by increased radiolunate angle (>15deg dorsal); typically associated with scapholunate ligament disruption, scaphoid fracture/malunion, or distal radius malunion; associated with characteristic pattern of wrist arthrosis (SLAC/SNAC)
  • Scaphoid flexes due to its attachments to trapezium, trapezoid, and capitate
  • Triquetrum extends and pulls lunate into extension
  • Radiolunate joint remains congruent until… end-stage SLAC (type IV) 
111
Q
  1. What is true with respect to acute traumatic rotator cuff tears?
  2. They are most often torn at the posterior supraspinatous
  3. The most common mechanism is fall on outstretched hand
  4. Mechanism is a traction injury of the arm/shoulder
  5. Associated biceps tear is not tear common
A

ANSWER: B

2019

  • Pathophysiology
  • mechanisms of tear includes
  • chronic degenerative tear ( intrinsic degeneration is the primary etiology)
  • usually seen in older patients
  • usually involves the SIT (supraspinatus, infraspinatus, teres minor) muscles but may extend anteriorly to involve the superior margin of subscapularis tendon in larger tears
  • chronic impingement
  • typically starts on the bursal surface or within the tendon
  • acute avulsion injuries
  • acute subscapularis tears seen in younger patients following a fall
  • acute SIT tears seen in patients > 40 yrs with a shoulder dislocation
  • full thickness rotator cuff tears need to be repaired in throwing athletes
  • iatrogenic injuries
  • due to failure of surgical repair
  • often seen in repair failure of the subscapularis tendon following open anterior shoulder surgery.
112
Q
  1. What is true inrespect to GT fractures of the proximal humerus
  2. 50% of anterior shoulder dislocations are associated with GT fx
  3. Less than 5mm of GT displacement is associated with good outcomes in non-operative treatment
  4. The most common displacement is anterior and superior
  5. Arthroscopic treatment is preferred
A

ANSWER: B

2019

A concomitant fracture of the greater tuberosity (GT) is seen in approximately 20% (range: 15.5–25%) of patients presenting with anterior shoulder dislocation 

113
Q
  1. Patient sustains stab wound to medial upper arm and a traumatic laceration to the ulnar nerve. Along with a direct repair of the ulnar nerve, what are surgical adjuncts should you consider?
  2. End to side AIN to ulnar nerve transfer
  3. Ulnar nerve to musculocutaneous nerve (Oberlin transfer)
  4. Brachialis to axillary ?
  5. Some other wrong answer
A
  • ANSWER: A

2019

Supercharged End to Side Transfer (SETS). Originally described for ulnar nerve injuries.  For nerve injuries in which incomplete regeneration is anticipated, a SETS nerve transfer may be useful to augment the regenerating nerve with additional axons and to more quickly reinnervate target muscle. We describe our technique for a SETS nerve transfer of the terminal anterior interosseous nerve (AIN) to the pronator quadratus muscle (PQ) end-to-side to the deep motor fascicle of the ulnar nerve in the distal forearm. In addition, we describe our postoperative therapy regimen for these transfers and an evaluation tool for monitoring progressive muscle reinnervation. Although the AIN-to-ulnar motor group SETS nerve transfer was specifically designed for ulnar nerve injuries
* Ulnar nerve to musculocutaneous nerve (Oberlin transfer) – Used commonly in obstetrical brachial palsy injuries a fascicle of the ulnar nerve (supplying the flexor carpi ulnaris muscle) is cut and sutured end-to-end to the biceps nerve in the upper arm

114
Q
  1. What is true with respect to calcific tendinitis of the shoulder? (of the rotator cuff?)
  2. The etiology is known
  3. Can be due to both a degenerative and reactive process
  4. It touches the bone
  5. It can be seen on Xray (calcium deposits) particularly in the acute and resorptive phase
A

ANSWER: D

2019

  • Calcific tendinitis should be differentiated from insertional calcific tendinitis, which is believed to be a degenerative process. Similarly, calcific tendinitis of the rotator cuff tendons is thought to have a different pathophysiologic mechanism than calcific tendinitis at other sites, such as the Achilles or patellar tendons. Although theories of a primarily degenerative etiology have been supported in the past, Uhthoff and Loehr described an active, cell-mediated process that is now widely accepted. Unlike previous theories in which a necrotic degenerative tendon eventually calcified, Uhthoff and Loehr described cartilage metaplasia as a reactionary process in an active biologic environment. They described three main stages of calcification: precalcific, calcific, and postcalcific. The calcific stage was subdivided into three separate phases: formative, resting, and resorptive
    • The typical calcific deposit will be approximately 1.5 to 2 cm from the insertion near the tendon’s critical zone, an area of decreased vascularity near the insertion of the superior rotator cuff15. The
  • calcification may be differentiated from dystrophic calcification, which occurs at the tendon’s insertion.
115
Q
  1. Trigger Finger Release, what is true? 
  2. There is a 20% complication rate 
  3. RA and non-RA patients have the same outcomes 
  4. You should not perform a percutaneous release at the thumb 
  5. A2 pulley
A

ANSWER: C

2019

The proximity of the radial sensory nerve to the A1 pulleys of the thumb and the index finger has prompted some authors to recommend that these digits not be treated with percutaneous re- lease. JAAOS.  

116
Q
  1. Regarding complete AC joint dislocations, what is true?  Numbered List Properties
  2. Capsule and AC ligaments prevents superior and inferior motion 
  3. The CC ligaments and capsule prevent anterior and posterior motion 
  4. They are associated with a higher incidence of SLAP tears 
  5. Grade 3 AC joint separations do better with surgery  
A

ANSWER: C

2019

https://oce-ovid-com.proxy1.lib.uwo.ca/article/00004623-201401010-00012/HTML

Acromioclavicular joint injuries often present with associated injuries to the glenohumeral joint, including an increased incidence of superior labrum anterior posterior (SLAP) tears that may warrant further evaluation and treatment.

117
Q
  1. what is true with respect to loss of motion with radius/ulna fractures? 
  2. Distal and loss of supination 
  3. Distal and loss of pronation 
  4. mid-shaft loss of supination 
  5. mid-shaft loss of pronation 
A

ANSWER: C

2019

Yes – loss of pronation more with distal #

But better answer is mid-shaft or prox loss of supination (more affected, sup barely affected by distal #)

118
Q
  1. Regarding acute AC joint injuries, what is true with respect to hook plate and non-operative management? 
  2. Comparable clinical outcomes at 3 months 
  3. Radiographically equivalent at 1 year 
  4. General health outcomes are the same 
  5. Same rates of late rates of re-operation 
A

ANSWER: C

2019

Hook plate fixation does not lead to improved general health status compared with Non-op treatment. Presently, there is no definitive evidence that hook plate fixation is superior to Non-op treatment for acute complete AC joint dislocations.

119
Q
  1. What is not a risk factor for re-dislocation following Bankart repair? 
  2. ALPSA lesion 
  3. >20% glenoid bone loss 
  4. Anterior hyperlaxity (ER > 85 deg) 
  5. Age < 20 
A

ANSWER: B

Looks like newer evidence suggests smaller percentage (16-17%) - ALPSA lesion a risk factor (however, only found in Boileau study)

Let’s all agree this question sucks

2019

  • AK – ALPSA not a risk factor???…Charles: agreed
  • Historically, 20% to 25% has been accepted as the “critical” cutoff where glenoid bone loss should be addressed in a primary procedure. Few data are available, however, on lesser, “subcritical” amounts of bone loss (below the 20%-25% range) on functional outcomes and failure rates after primary arthroscopic stabilization for shoulder instability……some studies says it is less then this
    • The critical size of a defect in the glenoid causing anterior instability of the shoulder after a Bankart repair, under physiological joint loading - 2019
    • This computational study suggests that bone grafting should be undertaken for an anterior osseous defect in the glenoid of more than 16% of its length rather than a solely soft-tissue procedure, in order to optimize stability by restoring the concavity of the glenoid.
  • ALPSA lesion The overall risk of recurrence in the pres- ence of ALPSA-type lesions (Fig. 4) was 12.6 % compared to a risk of 6.5 % for Bankart’s lesions (p = 0.03) [25, 28]

Critical Value of Anterior Glenoid Bone Loss That Leads to Recurrent Glenohumeral Instability After Arthroscopic Bankart Repair

Show all authors

Sang-Jin Shin, MD, PhD*, Rag Gyu Kim, MD, Yoon Sang Jeon, MD, …

First Published March 23, 2017 Research Article Find in PubMed

https://doi.org/10.1177/0363546517697963

Article information

Abstract

Background:

Generally, a glenoid bone loss greater than 20% to 25% is considered critical for poor surgical outcomes after a soft tissue repair. However, recent studies have suggested that the critical value should be lower.

Purpose:

To determine the critical value of anterior glenoid bone loss that led to surgical failure in patients with anterior shoulder instability.

Study Design:

Case-control study; Level of evidence, 3.

Methods:

The study included 169 patients with anterior glenoid erosion. The percentage of glenoid erosion was calculated as the ratio of the glenoid loss width and the glenoid width to the diameter of the outer-fitting circle based on the inferior portion of the glenoid contour. The critical value of the glenoid bone loss was analyzed by means of receiver operating characteristic (ROC) curve analysis. Patients were divided into 2 groups based on the amount of glenoid bone loss: group A (less than the critical value) and group B (more than the critical value). Patients evaluated their shoulder function as a percentage of their preinjury level using the Single Assessment Numeric Evaluation (SANE) score, and postoperative clinical outcomes were assessed with the American Shoulder and Elbow Surgeons (ASES) score and Rowe score. Surgical failure was defined as the need for revision surgery or the presence of subjective symptoms of instability.

Results:

The optimal critical value of glenoid bone loss was 17.3% (area under the curve = 0.82; 95% confidence interval, 0.73-0.91; P < .001; sensitivity 75%; specificity 86.6%). Group A and B contained 134 and 35 patients, respectively. Shoulder functional scores were significantly lower in group B than in group A (P < .001). Five patients (3.7%) in group A and 15 (42.9%) in group B had surgical failure (P < .001). The SANE score was significantly lower in group B (83.8 ± 12.1) than in group A (92.9 ± 4.7, P = .001).

Conclusion:

An anterior glenoid bone loss of 17.3% or more with respect to the longest anteroposterior glenoid width should be considered as the critical amount of bone loss that may result in recurrent glenohumeral instability after arthroscopic Bankart repair.

120
Q
  1. Regarding total elbow, what is true? 
  2. Semi-unconstrained is more common than unconstrained because of decreased stress on the implant 
  3. Semi-constrained vs constrained increase the pressures at the cement-bone interface 
  4. Unconstrained needs precise axis of rotation 
  5. Do not recall, but likely wrong answer 
A

ANSWER: C

2019

121
Q
  1. What is true about the treatment of the fracture below?
  2. An anterior approach should not be used
  3. It involves the capitellum and not the trochlea
  4. It involves the trochlea and not the capitellum
  5. It can be managed non-operatively
A

Answer: A (2017) - Repeat

A – True – lateral column or posterior approach

B – false - both

C – False - both

D - False

122
Q
  1. If you over stuff the radiocapitellar jt by as little as 4 mm when doing a radial head replacement, you can expect…
  2. Increased contact pressures
  3. Loss of pronosupination
  4. Increased dislocation
  5. Ulnar positive variance
A

Answer: A (2017) - Repeat

A – yes, >2.5mm it does

B – No - be careful not to “over stuff” the component into the joint as this can lead to loss of extension

C - No

D – No – it would be ulnar negative

  • Radiographic incongruity of the medial facet of the ulnohumeral joint was an unreliable indicator of radial head overstuffing. Radiographic gapping of the lateral ulnohumeral facet demonstrated sufficient sensitivity to diagnose radial head overstuffing when compare to the standard length implant radiographs. Visual gapping of the lateral ulnohumeral facet on the cadaver specimens reliably indicated radial head overstuffing and should be a useful anatomic feature to assess intraoperatively.
123
Q
  1. Dude with simple elbow dislocation. Reduced and stable but opens to valgus stress in extension. What do you do?
  2. Cast at 90 for 1-2 weeks
  3. Cast at 90 for 3-4 weeks
  4. Dynamic ex-fix
  5. Open repair of MCL
A

Answer: A (2017)

A – Yes for dislcoations

B – put in supination – thumb point away from injury

C – no it’s stable

D – Never the answer, MCL repair not relevant in this case.

124
Q
  1. Best track record for glenoid component in TSA?
  2. Keeled, cemented
  3. Pegged, cemented
  4. Porous ingrowth base plate
  5. Metal backed base plate
A

Answer: B

2017

peg design is biomechanically superior to keel design

polyethylene-backed components superior to metal-backed components

glenoid not large enough to accommodate both metal and PE

125
Q
  1. Humerus condyles with respect to shaft:
  2. 10 deg anterior, 5 deg internal rotation, 6 deg varus
  3. 10 deg anterior, 5 deg internal rotation, 6 deg valgus
  4. 30 deg anterior, 5 deg internal rotation, 6 deg valgus
  5. 30 deg anterior, 5 deg external rotation, 6 deg varus
A

Answer: C (2017)

A – not enough anterior

B – not enough anterior

C - yes

D – no external

The medial plate incorporates 6–8° valgus alignment for the carrying angle. As the articular surface of the two condyles are not in the same coronal plane, the distal end of the medial plate was rotated about 5° internally and bent 30° anteriorly in relation with the humeral shaft. 

126
Q
  1. 75yo woman with massive rotator cuff tear and pseudoparalytic shoulder as a result. Which would provide her with the best outcome in terms of function and pain?
  2. Pec major transfer
  3. Lat dorsi transfer
  4. Reverse TSA
  5. Rotator cuff repair
A

Answer: C (2017) – indication for Reverse TSA is that description

127
Q
  1. Glenoid neck fracture. What is false?
  2. Usually stable fracture in absence of other injuries of bone/ligament
  3. Non-surgical treatment has good outcomes
  4. Deltopectoral approach to access
  5. Need to ORIF if displaced 1cm
A

Answer: C (2017)

Displaced scapula neck fx:

  • With > 40 degrees angulation or 1 cm translation need ORIF

Except for anterior fractures, there is little evidence supporting operative treatment for glenoid fractures and late instability of the shoulder does not appear to be a significant complication of non-operative treatment of fossa fractures

Whereas neck fractures are often managed through a posterior approach.

https://www.ncbi.nlm.nih.gov/pmc/articles/PMC5467683/#:~:text=A%20fracture%20of%20the%20scapular,fall%20on%20the%20outstretched%20arm.&text=Depending%20on%20the%20severity%20of,glenoid%20body%20may%20be%20seen.

128
Q
  1. old lady with transverse, non-comminuted olecranon fracture. Treated with tension band. Shown image. Fails due to pullout at 3 months, why?
  2. Two weeks not long enough to immobilize this injury
  3. Wire was placed superficial to triceps tendon
  4. she was smoking like a French Canadian truck driver
  5. Passive range of motion leads to early failure with a tension band technique
A

Answer: B (2017)

Wires pull out. Old question, previous exam.

Need to place K wires under triceps or they have high likelihood of pullout.

129
Q
  1. What is true with regards to obstetric brachial plexus palsy?
  2. Minimal biceps flexion at 3mos is an indication of neurotmesis
  3. can perform direct repair in the setting of neurotmesis
  4. horners is indicative of a high palsy
  5. phrenic nerve indicative of root avulsion
A

Answer: D (2017)

A – False. Mimimal biceps means it’s still intact

B - irreparable

C – No. Low palsy

D – true.

130
Q
  1. Which of the following is the least desirable option for a 50yo active female with a 3 part proximal humerus #
  2. Closed reduction percutaneous pinning
  3. Mini open disimpaction and tension band
  4. ORIF with plate and screws
  5. Hemiarthroplasty
A

Answer: D (2017) - althought in a 3-part A probably isnt a good choice either… Function would be good with the hemi but longevity would be a concern…

131
Q
  1. Which of the following is NOT a predictor of redislocation following shoulder stabilization:
  2. Age < 40
  3. Hyperlaxity
  4. Hill-Sachs lesion seen on AP X-Ray
  5. Contact Sport
A

Answer: A

2017

132
Q
  1. Which of the following is NOT a risk factor for dislocation following reverse total shoulder arthroplasty:
  2. Subscapularis repair failure
  3. Tear of posterior rotator cuff (infraspinatus and teres minor)
  4. Obesity (BMI > 30)
  5. Prior history of dislocation
A

Answer: B (2017)

  • Early dislocation after reverse total shoulder arthroplasty.
  • Chalmers PN1, Rahman Z2, Romeo AA2, Nicholson GP2.
  • Early dislocations of the RTSA prosthesis were uncommon. The most common associated factors were a BMI >30 kg/m(2), male gender, subscapularis deficiency, and previous surgery
133
Q
  1. what are the best indications for ORIF of a proximal humerus GT fracture?
  2. 3mm superior displacement
  3. 5mm posterior displacement
  4. 45 deg rotation of anterior fragment
  5. 45 deg rotation of distal fragment
A

Answer: B (2017)

>5 mm of displacement in the general population or >3 mm of dis- placement in active patients who are involved in frequent overhead activity.

134
Q
  1. What is not a primary stabilizer of the elbow?
  2. Anterior band of MCL
  3. LUCL
  4. Radiocapitellar joint
  5. Ulnohumeral joint
A

Answer C (2017)

Still important – but secondary stabilizer vs others are primary

135
Q
  1. When comparing the results of 1- vs 2- incision distal biceps repair, which of the following is true?
  2. Radial nerve palsy is the most common nerve injury associated with both techniques
  3. Suspensory fixation has a higher complication rate than bone tunnels or interference screw
  4. Both techniques <10% risk of complications
  5. Heterotopic ossification is a common complication with a one incision technique
A

Answer: C (2017)

A - false

B – false – endobutton > suture anchor > transosseus > interference screw …this is failure strengths and not necessarily “complication rates” however…. Clay

C - true

D – false

  • The greater rates of complications occurred in the single-incision group at 28.3% (222/785) versus 20.9% (104/498) in the double-incision group.
    • Both over 20% minor and major from newer papers, but JAAOS 2010 - showed complication <10%
    • Comparing repairs: BEST Endobutton> suture anchors=>Trans osseus >interference
136
Q
  1. What is the best way to assess MCL rupture of the elbow?
  2. Moving valgus stress test
  3. Valgus stress x-ray - Inconsistent results.
  4. Standard MRI
  5. Tenderness over the medial epicondyle
A

Answer: A (2017) - think this probably still the best answer

MRI - “Test of choice” - tough call though, less sens, more specific than MVST (MRI arthrogram is the clear best)

  • “The moving valgus stress test, described by O’Driscoll et al,11 was found to be 100% sensitive and 75% specific for MCL injuries. The test is performed with the patient upright; the shoulder is placed in 90° of abduction and maximal external rotation with the elbow in maximal flexion. A constant valgus force is applied as the elbow is extended to 30° of flexion. A positive test must reproduce pain over the MCL between 70° and 120° of flexion, which is reminiscent of throwing between the late cocking and the early acceleration phases.”
    • Aside: Regan and Lapner reviewed the prone push-up and chair push-up test and demonstrated 87.5% sensitivity in individual test use and 100% sensitivity when both tests are used together
    • Magnetic resonance imaging is the test of choice for valgus instability. Nonenhanced MRI is 57% to 79% sensitive and 100% specific for MCL tears, whereas magnetic resonance arthrogram with saline or gadolinium is 97% sensitive and 100% specific.9,23 The AMCL is visualized on coronal T1 images as a thin band of low signal intensity along the medial joint line. Sprains appear as increased signal on T1 and T2 images, whereas complete ruptures demonstrate poor definition, redundancy, or an irregular pattern.
137
Q
  1. What will cause shoulder instability symptoms?
  2. Suprascapular nerve impingement
  3. Spinal accessory nerve impingement
  4. Biceps tendinitis
  5. AC joint arthritis
A

Answer: A (2017)

  • subjective shoulder instability
  • failure of the scapula to provide a stable platform for rotation of the glenohumeral joint 
  • Since the supraspinatus and infraspinatus are dynamic shoulder stabilizers, their weakness and fatigue may result in occult shoulder instability with associated tendinitis and/or bursitis
138
Q
  1. In suprascapular nerve compression associated with massive rotator cuff tears, the most common location of compression is:
  2. Paralabral cyst
  3. Suprascapular notch
  4. Spinoglenoid notch
  5. Transverse scapular ligament
A

ANSWER: B

2018

  • articles for both entrapment at spinoglenoid notch and suprascapular, but suprascapular more common
  • b/c tear of supra and infraspinatus causes retraction of and medialization of muscle fibers leading to teathering/sling which creates tension on the SSN
139
Q
  1. In RTSA for fracture, healing of the tuberosities helps with what motion?
  2. External Rotation
  3. Adduction
  4. Abduction
  5. Internal rotation
A

ANSWER: A

2018

-the goal of maintaining the GT is to help with ER and the goal of keeping the LT is to prevent excessive ER which leads to dysfunction

140
Q
  1. All of the following are important contributors to elbow stability EXCEPT
  2. Anterior band of ulnar collateral lig
  3. Posterior band of ulnar collateral lig
  4. Lateral ulnar collateral lig
  5. Ulnohumeral articulation
A

ANSWER: B

2018

141
Q
  1. Which of the following is a test for ulnar collateral ligament instability?
  2. Drawer test
  3. Pivot shift test of elbow
  4. Pain at end range of motion
  5. Moving valgus stress test
A

ANSWER: D

2018

142
Q
  1. Which portal is most likely to have a sensory nerve deficit in elbow arthroscopy?
  2. Anteromedial
  3. Posterolateral
  4. Direct posterior
A

ANSWER: A

2018

Ulnar most common overall, SRN 2nd

Distal AL portal = Radial (SRN) most common purely sensory

Proximal AM = Ulnar most common motor/sens

http://www.wheelessonline.com/ortho/elbow_arthroscopy

AM portal for MABCN if no other option listed.

143
Q
  1. A patient sustains the injury shown in this image (it was a single coronal slice of the C-spine with bright fluid beside it … I think it might have been a root avulsions…looked like lower cord area, about 2 levels). What is the best treatment?
  2. Tendon transfers
  3. Neurotization (nerve transfer)
  4. Nerve repair
  5. Shoulder fusion
A

ANSWER: B

2018

  • Neurotization (nerve transfer)
  • transfer working but less important motor nerve to a nonfunctioning more important denervated muscle
  • use extraplexal source of axons
  • spinal accessory nerve (CN XI)
  • intercostal nerves
  • contralateral C7
  • hypoglossal nerve (CN XII)
  • intraplexal nerves
  • phrenic nerve
  • portion of median or ulnar nerves
  • pectoral nerve
  • Oberlin transfer
  • ulnar nerve used for upper trunk injury for biceps function
  • Muscle or tendon transfer
  • indications
  • isolated C8-T1 injury in adult (reinervation unlikely due to distance between injury site and hand intrinsic muscles)
  • priorities of repair/reconstruction
  • elbow flexion (musculocutaneous nerve)
  • shoulder stability (suprascapular nerve)
  • brachial-thoracic pinch (pectoral nerve)
  • C6-C7 sensory (lateral cord)
  • wrist extension / finger flexion (lateral and posterior cords)
  • wrist flexion / finger extension
  • intrinsic function
  • technique
  • gracilis most common free muscle transfer
144
Q
  1. Which is true regarding simple elbow dislocations:
  2. Instability is a common problem afterward
  3. You should splint for three weeks before starting ROM
  4. Supinated rehab protects the LUCL
  5. Posterior capitellar impaction is often associated
A

ANSWER: D

2018

145
Q
  1. Regarding proximal humerus varus mal unions, what is true
  2. RTSA should only be used in those older than 70 with a full thickness rotator cuff tear
  3. Subacromial decompression has good results if tuberosity malunion is less than 1.5cm
  4. You should do a hemiarthroplasty and tuberosity osteotomy
A

ANSWER: B (Shitty Q)

2018

  • *poorly remembered
  • JAAOS 2014 – malunion
  • Subacromial decompression has good results if tuberosity malunion is less than 1.5cm – no reason to do anymore
  • You should do a hemiarthroplasty and tuberosity osteotomy – possibly best of these three
    • JBJS 2003 - Nonprosthetic Management of Proximal Humeral Fractures
  • Persistent pain due to malunion of the greater tuberosity can occur with as little as 0.5 cm of superior displacement. The symptoms result from subacromial impingement. Such subacromial pain can be treated with subacromial decompression if the displacement is <1 cm. If the displacement is minimal and the fragment is ≤1 cm in size, excision of the osseous prominence that is causing impingement followed by rotator
  • ORIF +/- osteotomy, subacromial decompression, and soft tissue technique
  • Indications
  • symptomatic malunion following
  • nonoperative treatment
  • failed internal fixation
  • anatomical requirements
  • adequate bone stock for fixation
  • preserved articular surface
  • intact blood supply to humeral head
  • techniques
  • humeral head deformities
  • minor deformity techniques
  • open/arthroscopic tuberoplasty
  • +/- acromioplasty
  • +/- capsular release
  • +/- bursectomy
  • severe deformity techniques
  • varus/valgus osteotomy
  • +/- rotational osteotomy and lateral plate fixation
  • treated with corrective osteotomy/fixation if patient is young or active
  • may be augmented with strut allograft for poor bone stock
  • greater tuberosity deformities
  • <1.5 cm displacement
  • arthroscopic subacromial decompression +/- rotator cuff repair
  • >1.5 cm displacement
  • open/arthroscopic tuberosity osteotomy +/- subacromial decompression
  • outcomes
  • complication rates associated with surgical management of malunions are higher than those associated with acute fractures
  • shoulder arthroplasty
  • indications
  • symptomatic malunion following
  • nonoperative treatment
  • failed internal fixation
  • anatomical requirements
  • inadequate bone stock for fixation techniques
  • articular incongruity, destruction or collapse (e.g. osteonecrosis or head-split)
  • compromised blood supply
  • chronic dislocation
  • techniques
  • hemiarthroplasty
  • total shoulder arthroplasty
  • reverse total shoulder arthroplasty
146
Q
  1. When doing a hemiarthroplasty for a 4 part proximal humerus fracture, what is an important technical consideration?
  2. Head should be 0.5-1.5cm above GT
  3. uncemented prosthesis should be used
  4. prosthesis should be 3.5cm above pec insertion
  5. prosthesis should be in 50deg retroversion
A

ANSWER: A

2018

  • Head should be 0.5-1.5cm above GT - 0.5-0.8
  • uncemented prosthesis should be used
  • prosthesis should be 3.5cm above pec insertion - should be 5-8cm
  • prosthesis should be in 50deg retroversion – 30 retro
147
Q
  1. What is true regarding olecranon fractures?
  2. If proximal ulna dorsal angulation is decreased, the patient will lose elbow extension
  3. It is sometimes acceptable to leave an articular gap or bone loss in comminuted olecranon fractures
  4. A tension band wire construct creates more compression across the joint surface compared to a pre-contoured anatomic plate
  5. After excision of the olecranon fragment, advancement of the triceps to the anterior bony surface will result in better extension strength
A

ANSWER: B

2018

  • If proximal ulna dorsal angulation is decreased, the patient will lose elbow extension
  • It is sometimes acceptable to leave an articular gap or bone loss in comminuted olecranon fractures
  • Some bone loss can be accepted in the bare area. Bone graft should be used to bring the proximal ulna out to length at the posterior cortex. A gap in the nonarticulating bare area will fill with fibrous tissue as long as the posterior cortex is rigidly fixed. To secure the repair, tendon-grasping sutures are placed in the triceps tendon and passed through bone tunnels in the distal fragment. Management of bone insufficiency of the olecranon is based in large part on biomechanical studies on the amount of bone required to maintain stability. An et al17 suggested that up to 50% of the olecranon can be removed without rendering the elbow completely unstable.
  • A tension band wire construct creates more compression across the joint surface compared to a pre-contoured anatomic plate
  • The mean compression for plating was 819 N (sd 602, 95% confidence interval (CI)) and for tension band wiring was 77 N (sd 19, 95% CI) (p = 0.039). The mean compression on the articular side of the fracture for plating was 343 N (sd 276, 95% CI) and for tension band wiring was 1 N (sd 2, 95% CI) (p = 0.038).
  • During simulated movements, the mean compression was reduced in both groups, with tension band wiring at −14 N (sd 7) and for plating −173 N (sd 32). No increase in compression on the articular side was detected in the tension band wiring group.
  • Pre-contoured plates provide significantly greater compression than tension bands in the treatment of transverse fractures of the olecranon, both over the whole fracture and specifically at the articular side of the fracture. In tension band wiring the overall compression was reduced and articular compression remained negligible during simulated contraction of the triceps, challenging the tension band principle.
  • After excision of the olecranon fragment, advancement of the triceps to the anterior bony surface will result in better extension strength
  • JAAOS - Triceps reattachment in this manner creates a smooth, congruent transition from the triceps tendon to the articular cartilage of the olecranon but decreases the moment arm and may result in greater extensor weakness.
148
Q
  1. All of the following are signs associated with cubital tunnel syndrome except:
  2. Froment sign
  3. Feinklstein sign
  4. Wartenburg sign
  5. Jeanne sign
A

ANSWER: B

2018

149
Q
  1. In RTSA, scapular notching can be decreased by which if the following?
  2. Inferior tilt of the glenoshere
  3. Using a size 36 genosphere (they gave this specific number)
  4. Inferior translation of the glenosphere
  5. Using a 155 neck shaft angle
A

ANSWER: C

2018

  • Most true, can argue A too for standard Grammont style (old) prosthesis that previously had non-lateralized glenospheres and neck shaft angle 155.
  • JAAOS 2019 – Scapular Notching in rTSA
  • Systematic review 2000 rTSA = Neck shaft 155 sig more scapular notching then 135
  • -scapular notching is decreased by inferior translation and lateralization and decreased neck shaft angel (120-135 degrees) and glenosphere >42
  • -Articles: Athwal et al 2014. Does bony increased-offset reverse shoulder arthroplasty decrease scapular notching?
  • -Athwal et al 2015 JSES
150
Q
  1. Type 4 Monteggia fracture shown (proximal third ulna, and anterior radial head dislocation), what is true? 
  2. It is critical to restore the radial blow 
  3. PIN palsy is common 
  4. HO is common 
  5. Open reduction of the radial head is necessary 
A

ANSWER: C

2019

  • Heterotopic bone was identified on the radiographs of forty-eight elbows (37%).
  • More severe heterotopic ossification was encountered in patients presenting with an associated distal humeral fracture, terrible triad injury, transolecranon fracture-dislocation, or Monteggia fracture-dislocation. Patients with an open injury, instability, severe chest trauma, or delay in definitive surgical treatment had a higher prevalence of heterotopic ossification.

PIN palsy – 10%